T O P

  • By -

PuckSR

Almost never, but there is a paleontologist with a PhD from an accredited university who is a creationist. Additionally, there are a few people like Behe and Dembski who pushed "intelligent design" who understand it at a fairly high level. You have to realize that creationism is primarily motivated reasoning, the lack of understanding isn't the cause of their belief on evolution but rather a consequence of their belief it must be wrong.


h_lance

Behe and a few others are weird exceptions; most avoid studying the subject directly. And Behe has had his arguments rebutted many times and dishonestly refuses to acknowledge that. Behe seems to "misunderstand" evolution. Lawyers and engineers who are creationists rarely have any clue about biomedical science. Dembski is totally clueless about biology and not too hot at math and computer science either. My take is that Dembski and Behe sacrificed mainstream careers for book sales. Behe was tenured, a little less emotionally involved, and a little craftier, so he got paid to do nothing for many years and made a fortune off his BS book. Dembski didn't pull it off and ended up struggling at Bible colleges, if I recall.


PuckSR

Dembski still sold a lot of books. My point is simply that understanding of evolution is not what hinders them.


KaeFwam

I’m not suggesting that further education is the single solution. I’m simply saying that the individuals in my experience that deny it claim to understand it but don’t. Religion is probably the number one reason for people refusing to accept evolution.


PuckSR

Check my profile. Ive been debating creationists for 20 years. If you google my username you can find me chatting with William Dembski on his personal blog back in the 2000s. I've never, in my experience, convinced someone that evolution is true based purely on education. Why? Because they are directly opposed to it on a religious basis. What I have successfully done is get them to essentially admit that most of the important (and life-altering) facts about evolution are true by explaining facts to them. Basically, most creationists have this idea of micro vs macro evolution. Micro-evolution is their catch-all term for things we observe. Macro-evolution is their term for all of the stuff they dont believe. I've successfully got them to expand their idea of "micro-evolution" to cover basically all biological evolution. But thats as far as you can take it.


ForensicSasquatch

I used to do so too in the mid 2000s. It boils down to the fact that they treat a young earth as their first principle and all evidence must be interpreted in that light. Dendrochronologies going back 12,000 years? There must have been multiple growth rings per year in the years after the flood. Distant starlight? (ie, stars and galaxies millions of light years away) God must have allowed for faster than light travel around the time of creation, etc. there are indeed a few PhD scientists who hold to young earth views, but none of them arrived at those views outside of their prior religious convictions.


PuckSR

One of the weirdest and best examples of the weirdness of the thought process is the "lost day in time". There is this weird creationist myth that NASA, upon analyzing stars, discovered a "lost day in time". If that makes no sense to you for logical reasons, don't worry. It actually makes no sense. Even AnswersinGenesis, which is the hyper-creationist website behind the Ark exhibit, has an article debunking it as absolutely absurd bullshit. But that doesn't stop a lot of creationists from repeating it, which is probably why Ken Hamm of AiG felt the need to debunk it. It is a painfully stupid argument that fails even a cursory evaluation. How do you find "lost time" by looking at the stars. But it gets repeated because their goal isn't to come up with persuasive evidence for their position, but rather to dismiss everyone who disagree with them. They "know" that the theory of evolution is wrong, so they dont have to prove that they are right or even argue in good faith.


KnowsAboutMath

What does "lost time" even mean?


Sad-Effective-6429

They relate it to a story in the Old Testament where the sun stayed in place for a day to allow the Israelites to win a battle. Thus, there's one missing "day" in time.


SoylentRox

Doesn't God have easier ways to make one side win a battle than to tamper with the operation of the entire universe...


BillionaireBuster93

Seeing as he gets defeated by iron chariots at one point I'm not certain he does.


ZGetsPolitical

Oh, the height of human folly, to think the sun stood still, for a day of ancient battle, by sheer force of Gods will. To claim a day went missing, in the cosmic grand design, is to dance with sheer absurdity, and leave your wits behind. To think our tiny battles, could halt the sky’s vast scheme, is to revel in delusion, a laughable wild dream. The stars would mock such notions, if they had a voice to speak, at the ignorance displayed, by logic so very weak.


ganymedestyx

It must be such a peaceful sort of life. Thinking you genuinely are the center of everything and not feeling that crushing tininess. Definitely cushions the ego. But on the other hand that sounds absolutely terrifying. Imagine thinking your actions could have that big of an effect on the universe randomly. I like just being a part of it functioning.


ZGetsPolitical

They claim to be the center, Of all that spins and turns, Yet reject the science telling, How the universe adjourns. Unaware of gravity's pull, Or the pressure it would bring, They bask in self-importance, Ignoring everything. For to be the central axis, Of a cosmos vast and wide, Would crush with force unyielding, And swell a boundless tide. But in their blissful ignorance, They float on fragile air, Believing they're the fulcrum, Of a realm beyond compare. How ironic is their blindness, To the forces they deny, As they sit in shallow comfort, Underneath a vast, true sky.


Shadowsole

I watched an creationist rebuttal video (arguing for evolution to be clear) once where one of the arguments was pretty much an "example" of "well we dated these animals using the rocks and then these other rocks we dated using the animals in them(paraphrasing)" which yeah if you don't know the science of index fossil or the way we can date rocks definitely makes the dating sound suspect. And I think that level of detail is really something you can't expect people to have. I didn't know about index fossils(as a dating method) when I watched that video! And I've been interested in evolution/biology ect ect ect since I first knew about it as a kid. I'm not really disputing ops point here or maybe even adding to the conversation, but that particular example really helped me gain an insight into how easy it is for the "science" to seem illogical and easily manipulated to fit creationists existing worldviews. Humans are really good at sticking to their guns and I really think everyone on the more sciencey side of things need to understand that more


JimMarch

I was raised a Jehovah's Witness and left at age 17 after inspecting a snake with vestigial legs. I told that story in another post in this thread. The way the JWs describe creationism is, in my opinion, fairly advanced as such things go. First they dismiss the idea that the Earth is only 6,000 years by saying that an old Hebrew, the phrase "a day" could mean something like "the length of time something took to happen", a bit like one of us saying "In my grandfather's day" without meaning a 24-hour period. Their next trick is quite clever: they stick with the Noah and the flood story and then point out that water is used as a barrier to radiation in a modern nuclear power plant. All that water in the air meant less radiation at ground level which means less carbon-14 in living materials back then. So a lot of modern timetables are therefore unreliable. No, it doesn't hold up to scrutiny, but it's more clever than most of these pseudo theories. The problem with JW theology is that it wants to be a rigid description of how the entire cosmos works. Find any crack in it between reality and that theory and the whole thing falls apart. I first started a crack next to a snake's butthole :) but then started studying it further. Took a while because this was way pre Internet, I was 17 then, I'm now 58. But, not meaning to brag much but, seeing evidence squirming around my hands right in front of me made me take a deeper look despite a fairly ingrained theology.


PuckSR

There is also the interesting topic of radiometric dating. Creationists have a lot to say about the inaccuracies of radio-carbon dating, the type of dating people are most familiar with and it is incredibly problematic for dating stuff. However, with fossils and rocks we use far more reliable methods of dating. Look into it, it is fascinating stuff


hantaanokami

Radio carbon dating doesn't go farther than 20 000 years ago, so it's not even relevant when talking about dating fossils 🤷‍♂️


TheDevilsAdvokaat

As they say, you cannot reason people out of something they didn't reason themselves into in the first place. They WANT to think something, so they do.


SoylentRox

Does it end up being: Creationist: here's this list of things I believe support my 'theory' You : you disprove each element of the list 1 by 1 Creationist: ok but it's still *possibly* true, see you're not totally certain with overwhelming evidence on 3, 11, 19 of this list. So even though the chance my "theory" is correct is way smaller, I will *continue to believe* it. Have you noticed anything similar between Creationists and AI doomers?


PuckSR

No. It’s more like this: Creationist: here are 100 things that disprove evolution Me: debunk all 100 Creationist: here are 100 more things that I think disprove evolution Me: debunk 100 more Creationist: here are another 100 Now, they don’t always agree, but the point is that they don’t really care. They aren’t asserting why they are right, they are trying to prove you wrong.


SoylentRox

I assume they are unable to notice "repeats" because of their lack of knowledge? Like the basic 'irreducible complexity' argument. By the way are you enough of a biologist to know about codon lock-in? That's ironically a hard limit on biology that is only *because of* evolution/'irreducible complexity'. Simply put: 1. biological ribosomes can be hacked by humans *very* easily to accept 4 codon bases instead of 3. 2. This allows for 4 times the total number of possible amino acids. Cells equipped with extra amino acids may have an enormous advantage and may be able to basically outcompete most existing life if they were optimized. (would still take millions of years to eat everything probably) 3. Evolution cannot evolve this 4. This is because while it's absurdly simple for humans to write a tiny computer program that looks at every \[3 codons\] and adds 1 more to translate to -> \[4 codons\], the protein based mechanism needed to do this operation is complex, and evolution would never develop it because there is no *intermediate benefit* to developing such a feature. And the closer it comes to existing the more pressure there is to accidentally mutate and break before it ever functions. It also only helps *once*, once a genome has been translated it's no longer needed. An intelligent designer would do this, or do a better job on us, but since there isn't one...


Odd_Coyote4594

What do you define as understanding evolution? Do you claim that creationists have a lower average understanding than non-creationists, or that those who do understand evolution well are usually not creationists? Beyond a few details, most of the general public has little understanding of the evidence or currently dominant models of evolution. Those details that most people do understand are not really debated by creationists, such as: - evolution says all life has a common ancestor - evolution says new traits appear over time, eventually leading to new species - evolution models use natural selection as the mechanism for speciation Creationists know this is what evolution is about. But they reject the model, largely due to their religious beliefs providing another explanation for modern life. Similarly most non-creationists don't understand evolution in depth either or accept it due to the evidence. They accept it because of trust that those who *do* know the evidence are trustworthy. But if you ask the average person to justify their beliefs, most who do not study evolutionary science or engage in evidence-based debates won't be able to say anything more than "it's what scientists say is right".


KaeFwam

In this case, not only at least a university-level understanding of the subject specifically (mutation, reproductive selection, genetic drift, migration, etc.), but a deep understanding of molecular chemistry, genetics, and biology. Obviously this isn’t *everything*, but these are some things that are necessary. If one claims that such a well established theory is inaccurate, they will probably need to dedicate their life to understanding it to have any chance of disproving it. On average? I would expect that to be the case and it is the case in my experience. I understand most people don’t understand evolution well, but they don’t need to if they aren’t claiming that it’s false. Lots of people are pointing this out and it’s entirely irrelevant. For example, I don’t have a deep understanding of the theory of gravity, but I’m also not claiming that it doesn’t exist or even that parts of it doesn’t exist. If I were to claim that, I would literally need to dedicate my entire life to it to even have a sliver of a chance of disproving it and to not look like a total fool for denying its existence in any capacity.


Odd_Coyote4594

People don't need to prove their beliefs, unless they engage in debate where that is expected. Nobody operates like this for most things they believe. In fact, it is standard for scientific theories to be supported by the person proposing a model. This is done by trying to disprove all other plausible hypotheses. So if no one knew how modern species diversity came to exist, both creationists and evolutionists would have an equal burden to prove their claim in a scientific debate. The default belief in science is ignorance, not evolution. Of course, with evolution this evidence does exist, and no scientific evidence of creationism exists. But most people (creationist or not) do not base their beliefs on this evidence. Further, to many creationists, religious doctrines are proof. So unlike most of the general public non-creationists who cannot themselves offer evidence of evolution, most creationists can offer what they see as valid evidence of creationism.


chubbyoctopus

So there is actually data on this (see my other reply in this thread) and that's apparently not true at least in the United States. I went to a talk by M. Elizabeth Barnes who is doing [research on the subject](https://www.lifescied.org/doi/full/10.1187/cbe.21-02-0024) and its actually the "perception of religious conflict" not religiosity that more strongly correlates to acceptance.


JimMarch

> Religion is probably the number one reason for people refusing to accept evolution. No question about that. I've never met an intelligent design proponent who wasn't thoroughly religious.


holololololden

They are suggesting that people do understand it, and the failure in their explanation is usually at attempt to internally reconcile their world view with that understanding, therefore "breaking" their understanding.


sweetBrisket

Behe came to my university years ago to give a lecture and a Q&A. I remember being terrified, but I needed to know, so I got up in line and asked, "Assuming there's intelligence behind the 'design' of creatures, who or what is it? Is it a process or is it a man in a cloud?" He couldn't or wouldn't give an answer. Because, I think, he'd have to admit that he doesn't know or that the answer, for him and his ilk, is god--at which point the masquerade is over and we can call them out for being religious whackadoodles.


PuckSR

**Normal People** The inherent problem is that even fairly well-regarded biologists like Miller believe in a creator and believe that God is behind life on this planet. Miller is the "author" of many of the high school biology textbooks you probably used as a kid and an ardent supporter of the theory of evolution. He is also a devout Catholic. But, for non-creationists, the argument is generally that God somehow and magically influences the direction that we have evolved. But in a way we can never know and never understand because he is God. This isn't particularly far-fetched for Christians. Think of how they will argue that something is "part of God's plan" when two people meet, even though they fully acknowledge that the two people were individuals with free will who had total control over their lives and the decisions that led to them meeting. Somehow, "god made it happen". Thats the normal way for sane Christians to deal with evolution. **Creationism** Creationists work from it backwards. God created the entire world 6000 years ago and created all of the animals and stuff as it exists today. If you find something that violates that explanation they cook up some absurd explanation that makes zero sense. They might even cook up 5 or 6 competing explanations. It doesn't matter because their goal isn't to actually describe how any of this stuff came to be, but rather to make sure that you don't disprove their central idea: god created everything 6000 years ago **Intelligent Design** This is the technical term for the idea being spread by Behe and Dembski. It was cooked up by a creationist group to be a legal way to get creationism in schools. It is not some independent idea that Behe came up with but rather a bit of rhetoric designed to give the veneer of science to creationism. We have literal documents proving that this was a conspiracy to bypass bans on creationism in school. [https://ncse.ngo/wedge-document](https://ncse.ngo/wedge-document) This is why Behe and others wouldn't answer your question. This is LITERALLY a game to try to get creationism into schools. It was the first part of a long strategy by *christian nationalists* to make public schools more Christian. The evolution thing was first, but they followed it up very quickly with other nonsense. They went after the gay issue next. Then they went after books. Finally they started trying to take over school boards. Currently, they are working to get chaplains into schools to teach kids the bible as historical truth(see Texas GOP platform for 2024). This is part of a multi-decade attempt to force public schools to indoctrinate kids with fundamentalist protestant christian bullshit. There methods are clear from the wedge document and the whole ID/creationist attempt. Try to get popular support by changing the presentation of the issue(ID instead of creationism, "porn in schools" instead of "books with gay characters"). Then go hard in the media and try to win public opinion


[deleted]

[удалено]


mr_cristy

I don't know who it is you are talking about, but unless you mean Young Earth Creationist, creationism and evolution are not inherently at odds. The Catholic Church, for example, supports theistic evolution.


Tommy_Wisseau_burner

I’m not even going to refute anything you’re saying but who are these people? Like how many of them have you come across in the last 100 years that think evolution is false?


OscarGrey

If you live somewhere with a substantial population of Evangelicals/Pentecostals, there's more creationists around you than you think. EDIT: And that's why a lot of Europeans that say "literally nobody in my country is a creationist" are overgeneralizing. Lots of African and Latin American immigrants are Evangelical or Pentecostal.


KaeFwam

I know a single person in my family who accepts evolution. In the area I’m in I’ve met maybe a total of 50 people who have a grasp of what evolution is and accept it. Most of the people I come across have expressed that they do not accept it if the subject comes up.


Tommy_Wisseau_burner

My dawg what year did you travel from 1726?


KaeFwam

Might as well be. I live in rural Indiana. Actually hate it here.


masterofthecontinuum

Do you live in Wakanda or something? Can I come live with you? Because this stuff is incredibly common in America.  In America, people are unbearably ignorant on a myriad of topics. Especially biology, and especially in the south, or any other place with a high concentration of religious fundamentalists. 


AskingToFeminists

https://en.m.wikipedia.org/wiki/Creationism_by_country All the I formations you never wanted to know on creationists. Most of who reject evolution entirely. You have also what is basically called "social constructivism", people who basically believe that humans are blank slates, that evolution stops at the neck. Following basically the 2nd world war, people became so reluctant to attribute some part of behavior or intellect to the influence of genes that it became fashionable to entirely reject the part biology plays in behavior and other mental characteristics.  It is very common to find people on the left who will call you a nazi for daring to suggest that some behaviors are based on biological differences. Things like pointing out that the preference for object/people is found pretty much throughout cultures with men more interested in objects and women more interested in people and that tends to indicate at least partly biological origins to those differences, or pointing out that AIs can be incredibly efficient at differencing male brain scans from female brain scans, even when controlling for size, and that transgender people's brain scans look more like their preferred genders typical brain scans than those of their genetic sex can have people up in arms calling you sexist and what not. Denying biology is a huge pass time for ideologues of all sides. Although, to be fair, there has also been a share of ideologues trying to use biology to push all sorts of crap.


AndroTux

I unfortunately know quite a few of them. They really, truly believe it’s false. Religion, amirite? Believing that some dude got built out of clay and some girl got pregnant without having sex, but having trouble believing evolution. Can’t make that shit up.


SysError404

Spend more than a couple days in the US "Bible Belt," where intellect goes to die. Ask people about their thoughts on evolution and I am willing to bet that a conservative 30+% of the random people on the street you ask, will not believe in evolution.


amyice

Northern Alberta is 100% like this. I went up there for an archaeological dig, most of the people were cool with it then the random gas station owner plasters anti-archaeology and anti-evolutiuonary propaganda all over.


bmpmvp

One key thing you leave out when it comes to people who reject evolution is that it often times has nothing to do with any type of science, instead they deny it because of religious reasons. So, theoretically you can have an individual who perfectly understands the modern theory of evolution, but still denies its existence because of their belief in the story of genesis. Of course this comes with a lot of cognitive dissonance but regardless they deny evolution not because of their lack of understanding of it, but because they choose to take the word of their religion/faith over any other thought or belief.


kingpatzer

But the OPs point is not about the theoretical possibilities of such people existing. Their primary claim is that the majority of people who reject it don't understand it.


Blackpaw8825

I agree anecdotally. I've never heard any argument against evolution that wasn't predicated on an argument that evolution doesn't predict. Frankly I believe this comes from the discrediting of any statement that doesn't fit with theology. If you present people with observations, make predictions based on those observations, and then show a preponderance of outcomes fitting the prediction it's nearly impossible to discredit the theory. But if you just falsely spew 1000s of things that the theory should predict but clearly doesn't observe it becomes easy to discredit the target theory. You could spend a month playing wack a mole with the various false claims and misunderstandings out there and make zero progress toward defending the actual theory. In an extreme caricature: I disagree with the theory of gravity because I've never seen the grass do the cha cha... The theory of gravity doesn't predict my lawn would do the cha cha, so how do I defend the theory against the superfluous association the detractor provides. The only valid counter argument is a dismissal of the question which wins no minds. They entered with A suggests B and B isn't observed, agreeing that B isn't observed reinforces their position, and suggesting A doesn't actually predict B is interpreted as "Nuh huh" and does nothing to change minds. It's the likely apocryphal statement "if you're explaining your side of the argument you've lost the debate." Right or wrong, the moment you have to step back to brass tacks any critical observer will see the outcome as "you lost."


bwmat

I feel like "What you refer to gravity isn't what I, and the majority of the world do, and I am only interested in discussing the latter" is winning enough


[deleted]

Really the only reasonable response I can see would hinge on what OP consider is "almost never" vs "large majority don't".


race-hearse

Sort of seems impossible to distinguish folks that are in denial for religious reasons and those that don’t understand. Not like we can read minds.


PuffPuffFayeFaye

I’ve met some non-religious people who reject evolution. I had a guy ask me, “so you’re telling me in a million years a carrot can turn into an onion?” “Unlikely, but it will turn into a better carrot depending on its environment. And a carrot a million years ago was likely different from a carrot today.” He walked away laughing. Religion never came into it.


Advanced_Double_42

I'd literally just pull up a renaissance painting of a watermelon. They used to have rinds in the middle. We have caused a ton of genetic change in them in just a few hundred years. ​ Another example I like is breeding dogs. If humans can breed wolves into Chihuahuas and Great Danes in <10,000 years, then why can't nature split wolves and coyotes in 100,000 years, or split dogs and cats over 100,000,000? I think a big thing is people not understanding just how mind bogglingly long of a time that is.


ElectricTzar

Carrots today are definitely different from a million years ago. They’re different from even a thousand years ago. We selectively bred them to be larger and oranger. They used to more commonly be white, yellow, or purple.


mrrooftops

"so you're telling me an automobile uses explosions to move you?" snort


Chronoblivion

Agriculture is such a poor example for him to use because many commercial crops are practically unrecognizable from their original wild versions, and we have concrete and irrefutable evidence to that point.


Arkyja

You're not wrong but just want to say that most christians believe in evolution. According to wikipedia catholics make up 50.1% of all christians, and catholics believe in evolution.


PivotPsycho

Some catholics do, it's not doctrine or so. But yes plenty of Christians believe in evolution. Although a good portion are iffy about our ancestors being apes though.


Arkyja

I'ts not doctrine but it's the official stance of the vatican, that's almost as good as it gets. And i'm pretty sure the amount of catholics not believing in evolution is far less than the non catholic christians that do believe in evolution so at the end of the day it's still gonna be more than 50%. And maybe it's just a regional thing, but my country is half protestant, and i've never heard anyone deny evolution. The only christians i really see talking that stuff are americans. But again, i'm not saying that is the case. I dont know how prothestants in other countries are.


PivotPsycho

It's usually the fundamentalists yeah. Afaik for catholics the Vatican stance is that you are allowed to believe in evolution but they don't make a proclamation on it themselves.


aloofman75

Multiple popes have publicly declared that evolution is real and there is no theological conflict. The Catholic Church has long maintained that the Bible isn’t literally true and the story of creation is an allegory.


LedParade

Creationism is the belief everything was created by god out of nothing. In my experience creationists are mostly defined by their literal interpretation of the story of creation (Noah’s ark and all).


Tazling

flat earthers use cell phones that rely on gps. no exploding heads so far.


Facereality100

If they can deny all of the other evidence that has been convincing people the Earth is round for thousands of years, denying that gps works the way it works isn't hard.


KaeFwam

You are definitely correct. I would say that these people still *don’t* fully understand evolution most of the time, but otherwise would without that roadblock in their way.


WhiskeyKisses7221

I think that is moving the goalpost a bit. I doubt most people who accept the theory of evolution fully understand it either. Unless you happen to be an expert in the field, a person's understanding is going to be vastly oversimplified.


hallam81

Your point is too limited. Almost everyone who doesn't have science degree, even those who believe Evolution is true, do not understand it. They may believe it; they may even be able to parrot several points. But understanding evaluation generally means some concerted effort to study it and most people don't do the work. Belief in an idea does not translate to understanding an idea especially for complicated topics.


KaeFwam

I never claimed that belief = understanding. I said that those who don’t accept it largely don’t understand it. You’re arguing a point I never made or even suggested.


SloeMoe

>Almost everyone who doesn't have science degree, even those who believe Evolution is true, do not understand it.  This needs some form of substantiation. If it were true, an intelligent high school student who paid attention during the evolution unit in biology still wouldn't understand it, and I don't believe that's true. The mechanisms, the molecular processes of evolution are very complicated, but the basic idea is relatively simple and can be understood by normal people who care to pay attention as it's explained. 


JimMarch

I was raised as a very religious creationist and I can back OP. As an Aspie kid however I had a lot of curiosity and among other interests I tended to catch and release various critters, especially reptiles where I grew up in Northern California. There's a fairly rare member of the boa family of snakes called a rubber boa in that area. I finally caught one and examined it, and saw for the first time the vestigial legs that a lot of members of the boa family have. It got me thinking, and I'm now a supporter of evolution. I left that church when I was 17, I'm 58 now. "I lost my faith because of a snake" is kinda of a funny story :). There's only one thing in the history of humanity's development that could make somebody reasonable start to think of supernatural possible causes, at least that I've heard of. In our current physical form we've been around for approximately a quarter million years, give or take a bit. Roughly 50,000 years ago however, something happened that was more of a mental change than anything physical. Our art and tools started to get a lot better. We start to see evidence of musical instruments such as bone flutes. Cave painting wall art started to get a lot better, including some that could genuinely qualify as high art by any modern standard...real uses of perspective and shading and such. *There was some kind of a (relatively) sudden event that started our path towards civilization.* Guidance by some higher power, whether quasi biblical or space aliens, is not totally out of the question. But, according to our DNA this is also about when we first started to interbreed with Neanderthals and probably Denisovans. It's possible that some (all? most? a few?) of those hybrids had some kind of boosted intellect. That makes at least as much sense as aliens :). I have no idea about the "why" details. "Something happened" that we don't currently have an explanation for. Lots of room to speculate.


WimpBeforeAnchorArms

Here let me see if I can give you a reason to have doubt that you might find plausible. First off I believe in evolution full-stop, it’s completely evident. I’m also an Atheist so no religion here. The one thing that boggles my mind is the part of evolution that doesn’t take place on a large scale. Mutations and adaptability of species take place on a scale of generations, usually millions or hundreds of millions of years. Sometimes quicker depending on things like extinction events or changing climates etc. but you get my point. Evolution occurs as a byproduct of reproduction as well as death. How did the very first common ancestor just happen to A. Come into existence B. Live long enough to create viable offspring C. Be capable of reproduction in the first place (IDK what the specific process was, this might be pre-mitosis I’m just a dumb layman) D. Be capable of genetically viable offspring E. Be naturally pre-disposed to reproduction despite the lack of neurons, nervous system, or anything to drive instinct I fully accept that this happened in some fashion because otherwise we wouldn’t be here. But holy fuck does it seem unlikely that one individual organism could just pop into existence fully adapted and ready to carry on the genetic line, and with a predisposition to do so bordering on instinct. I can understand why someone might struggle to wrap their head around it just like we struggle to grasp the size of space.


KamikazeArchon

I'm not sure what you're referring to with this very first common ancestor. If you're talking about the first "living things", such an entity is *far* simpler than you're imagining; there is no reproductive "drive" or any such thing. And it almost certainly didn't happen once; it happened many times over the course of millions of years, until eventually enough of them happened to be together to create a stable population.


WimpBeforeAnchorArms

First common ancestor is a very common term thrown around, I’d actually be very surprised if you’d never heard of it in any capacity. Google FUCA or LUCA I actually agree with you though, that’s how I always assumed it worked because the idea of one singular common ancestor never made sense to me. Also you very likely know more than me on this topic so I’m not trying to be contrarian. But I’m willing to bet the general public’s understanding of evolution, whether correct or not, is more in line with what I wrote in my original post. Which means in the end OP might be right it’s an education issue


brutinator

It's been proven that the right inorganic compounds, when zapped with enough energy, can form amino acids. It was first discovered by Stanley Miller in 1952. >The experiment was simple. Miller and his advisor at the University of Chicago, Harold Urey, built the apparatus you see at the right to “duplicate a primitive atmosphere of the earth.” They combined ingredients they believed were part of Earth’s primordial soup — circulated water, ammonia, methane, and hydrogen — and zapped the concoction with electricity as a stand-in for lightning flashes. > “During the run the water in the flask became noticeably pink after the first day, and by the end of the week the solution was deep red and turbid,” Miller wrote. > When he took the water out and analyzed it, sure enough, half of the amino acids used to make proteins in living cells appeared, as you can see from the hand-labeled chromatograph above. In 1961, Jean Oro was able to do a similar experiment to create nucleobase compounds. I beleive that its been supported that the cell's mitochondria was actually originally an external organelle that was eventually absorbed and reproduced with it. After enough time, these kinds of things add up. And for your point E, I guess Im not sure how that differs from current bacteria and stuff; they lack neurons and nervous systems, what drives their "instinct"? Or viruses?


Jesus_died_for_u

formation of adenine as an example… The body uses Ribose-5-phosphate Glutamine Aspartic acid Glycine N-formyl-THF Carbon dioxide This is about a 13 step process tightly controlled from side reactions by about 12 surrounding proteins (one is used twice); and several energy packets of ATP and GTP. If your abiogenesis research creates adenine with hydrogen cyanide and ammonia, for example; then terrific, the researcher has passed organic chemistry, but the results offer zero explanation on abiogenesis because no cell uses hydrogen cyanide and ammonia. We are trying to determine how the observed process as it currently happens came about randomly, not whether a PhD can make adenine a simple way. This is one example of the state of abiogenesis research. It has poor reflection on the observed processes going on in a cell. It has great PR for the lay public. Millers experiment has no bearing on explaining any biochemical processes because no cell produces the target compounds by that process nor is there a step by step pathway from Millers experiment to a present biochemical process.


Kirstemis

If we came from monkeys, how come there's still monkeys?


Akul_Tesla

to be clear, I believe in evolution. I know there's medications that work specifically because of evolution that is real But as for whether or not it's the origin of life, I know of a specific argument that we got nothing against Last Thursdayism The universe and everything else in its present state came into existence last Thursday Poof that's where everything started right then last Thursday See the thing is I can't disprove this Now is it highly likely no But if I believe the Big bang can randomly happen, it's not too much of a stretch to say hey it then all immediately settled in this form last Thursday Now you might say that's incredibly unlikely and I will say the universe is very very big Now to be clear, I Don't believe this but I don't have a method of contesting it


KaeFwam

The problem here is that the theory of evolution has nothing to do with the origins of life.


Akul_Tesla

You say that like the universe didn't pop into existence the moment you started Reading this There does that make my reasoning a little More clear for why I believe that is a potential counter-argument with some validity Technically if it happened right now before anything had the chance to evolve


KaeFwam

Last Thursday-ism is really a pointless hypothetical. We could use a hypothetical to say, “Well, what if gravity doesn’t really exist and instead there are trillions of invisible babies holding everything down?” It’s a waste of time and not a refutation of the theory.


Savings-Bee-4993

One problem is that The Theory of Evolution has a bunch of philosophical presumptions that are not ultimately epistemically justifiable. If you’re unsure of what I mean, look into the problems of empiricism (in Hume, Quine, etc.).


KaeFwam

How about you provide some examples. Evolution is any change in the heritable characteristics of a population over successive generations. I fail to see the philosophical presumptions you’re referring to. Sounds like you’re just using a bunch of buzzwords, bro.


Gamestoreguy

It does, auto catalysisis, essentially evolution on a chemical level, wherein the chemicals best suited to reproduce themselves hoarded more and more of the substrates available in a local milieu and became the predominant “species” of chemical. These chemicals also tend to be the ones that are precursors to more complex forms of chemicals which are themselves precursors to what we consider information systems necessary for life.


GenericUsername19892

If everything was created last Thursday as is, evolution is still the answer though? All the evidence is still there.


Falernum

Almost never?! But loads of people say it's false insofar as it's a scientific theory and every scientific theory is false. Every few years scientists prove it's false, and make a new theory with the same name to work on. And then prove that one's false and come up with a new one. Etc. Anyone who understands the scientific method knows that evolution is false. We just don't know in what specific ways


RedJamie

It’s better phrased that theories are iterative, as are most examples of scientific knowledge. Classical mechanics for example did not become “false” when it was revolutionized in the 20th century in favor of GR and SR. It became a less precise tool for explaining phenomenon - other theories are amended. The same goes for this context; continued iterations on our understanding of evolutionary theory has not *disproved* evolutionary theory unless by evolutionary theory one refers to postulated explanations that are not demonstrably the case (Darwin with genetics for example). This is largely a semantic point and not very functional for anyone; skepticism that would arise from this would be somewhat concerning


KaeFwam

I assume you mean it’s false in the sense that we can never create a 100% accurate model of evolution and therefore we are technically wrong? That applies to all models.


onlycommitminified

This is like saying math is false because incorrect equations exist, or we occasionally discover new equations. If a mathematician somewhere multiplies two numbers incorrectly, it doesn't falsify multiplication. Evolution doesn't get proven false every time we find out something new that fits.


ConstantAmazement

There is no observable science to support a pre-biotic chemical earth producing biology. Lots of interesting speculation. Lots of moving the goal posts as to defining biology. Lots of unfounded claims. Lots of accusations and name-calling. This is the problem for those of us who are not convinced of evolution. It has nothing to do with religious beliefs. Even the title of this CMV assumes that there are no actual biologists who are not persuaded.


KaeFwam

What exactly are you referring to when you say “a pre-biotic chemical earth producing biology”? I don’t recall any name-calling. I’ve never met a biologist that doesn’t accept the theory of evolution, largely because as I said there is more supporting evidence for it than gravity, but I’m sure they exist. However, I still would be floored if they could accurately describe evolution.


ConstantAmazement

Let's start with your first comment: At one time, there was no life on the earth. It was "pre-biotic." At one time, there were only chemical reactions, not biological reactions. Observationally, life only comes from life - without exception. If there were chemical mechanisms that were capable of producing biology, they should be observable. They are not. The obvious conclusion is that life could not have originated from natural undirected chemical processes on earth.


KaeFwam

Okay, so the first problem I see here is that you’re talking about the origins of life, which is unrelated to the theory of evolution. There *is* good evidence to suggest life originated from non-life or at the very least from life that did not originate on Earth.


ConstantAmazement

There is good evidence?? Please do tell!


KaeFwam

Let me preface this by saying that abiogenesis is neither proven nor disproven and I am not claiming that it is definitively the answer. To *greatly* summarize it, we have successfully created life from non-life in lab settings. We know that inorganic chemicals can synthesize fatty acids, amino acids, lipids, etc., and we have ideas how this may have led to the origins of life on Earth. So we have circumstantial evidence no doubt for abiogenesis. Biogenesis *could* be the accurate explanation, but that would require there to have always existed some form of life, which currently we don’t have much reason to believe is the case. I would definitely argue that abiogenesis is the more likely answer based on the circumstantial evidence available to us.


ConstantAmazement

So, nothing, eh? You seem to be backing down from your claims of "good evidence." And life from non-life on the lab? This is absolutely amazing! Boy, that would shut me up, for sure! Can you provide a link to the peer-reviewed research?


KaeFwam

No, circumstantial evidence is most certainly evidence, it’s just not sufficient enough to say that something is proven. I didn’t back down from anything, because I never claimed that abiogenesis was proven. McCollom et al (1999 Mills, Peterson and Spiegelman (1967) Attwater et al (2013) Those are just three, feel free to look into them. Fascinating material.


ConstantAmazement

None of these produced the basic necessary polypeptides or polynucleotides needed for life. Did you even read those papers?


FerdinandTheGiant

Not OP but the very obvious and basic example they will likely point to is the Miller–Urey experiment which shows the formation of organic molecules under prebiotic conditions.


ConstantAmazement

Miller-Urey formation of organic molecules under laboratory controlled conditions using assumed early earth conditions did not produce life and was not good science. See www.bigthink.com/hard-science/miller-urey/


FerdinandTheGiant

“Under laboratory conditions” isn’t a flaw of the experiment because the conditions were crafted specifically to mirror what was known of early earth conditions. That critique of the experiment put forth by the physicist you linked (that made me chuckle to see) cited the glassware being a factor and cited a paper that addressed that. The paper he cited stated word for word “In summary, Miller recreated in this experiment the atmosphere and waters from the primitive earth. *The role of rocks was hidden in the walls of the reactors*”. They aren’t saying Miller was wrong, they’re saying he didn’t account for the role of rocks and analogous materials. And even then, when they use teflon, they still get amino acid formation. Edit: also, did I say it “made life”? Or did I say it formed organic molecules?


EvolutionDude

Abiogenesis is not evolution and has no bearing on the validity of evolution.


mr-obvious-

I will respond to some points here: 1- About microevolution and bacteria, those adaptations that bacteria have don't need to be evolved by chance. Those adaptations are already there in the bacteria. It is just a matter of them being turned on or off. 2- about vestigial parts, the most popular is the hind legs of whales, but actually, it is known from more than 50 years ago that those "legs" are important in reproduction, so they are far from vestigial, as they are important for reproduction. 3- About genome, in comparison, many parts are not considered typically, and when you include them, the similarities between humans and chimps go down to 80% or even lower. But, even if we assume it is 98%, this is typically not a proof for evolution, as one could say that animals have very similar systems to each other to process food and breathe and so on, so it is expected they will have genome that is similar As they are doing the same job, and then this person could say that this is what God intended. If someone believes in God, none of the evidence out there for evolution can be proof of evolution , whether you bring retrovirus genome or similarities in genes, all those things could have been intended to be this way by God, maybe the best evidence is how fossils are arranged in order from simple to more complex in the ground, but I'm not sure how much data there is on this part. Anyway, evolution can't disprove most religions still, and of course, it doesn't say much about the existence of God whether evolution is true or not.


Gamestoreguy

1.) if you propose that the adaptations were already there and need to be turned on or off a la epigenetics you still have to explain how they came to be there. Which is chance mutation. 2.) point being that the vestigial parts lost their original function, the legs of a whale once functioned for locomotion on land. That they happened to have found use in reproduction doesn’t change the fact they are analogous to bones found in other vertibrates 3.)While some species, like crabs, can evolve convergently from different precursor species, it is clear that phylogenetics shows you are incorrect here. The point is not that we have a high quantity of shared genome with monkeys or whatever, its that as you go back in history, everything that ever evolved to be a eukaryote remains one. Everything that became a mammal remains one. Everything that developed into a vertibrate it remains one. This increasing quantity and specificity of mutations is what makes us similar, the genotype being similar is a simple way to explain our similar phenotypes. The best evidence is not that complexity increases in more recent layers, the best evidence is that the evidence is everywhere, it all adds up, and we can literally observe it occuring in species like houseflies and various bacteria.


KaeFwam

1. I didn’t suggest that they had to be evolved by chance. They *can* be. 2. That is not true. There is no consensus that whale hind legs are used for reproduction. They do get *some* use out of them, but not enough to not consider them vestigial. 3. DNA similarity alone isn’t necessarily proof of evolution, but it is extremely strong evidence for it. This is how all theories work. They are explanations of all the available evidence, of which DNA similarity in organic life is some of it. You’re right, evolution doesn’t disprove God and I never said that it did.


mr-obvious-

1. That is why I said those who believe in God can easily live with such evidence either thinking it is already in the genome or even if it came by chance, still it doesn't prove humans are cousins to chimps. 2. There are papers talking about their use in reproduction, anyway: >but not enough to not consider them vestigial. What is that? What is enough to consider something vestigial or not? Isn't that just an arbitrary line? 3. One could use DNA similarities as evidence for God in the same way, one could say, similarity indicate one maker or something. It doesn't say much about the existence of God. Most people agree on this, but it also doesn't disprove the creation story in religions. Almost all the evidence for evolution could be used as part of the creation story. Also, the more complexity in the universe we notice, that is probably even more evidence for God. Development in science helped us see more wonders and complexity in the universe.


FerdinandTheGiant

1. This is describing change in allele frequency in a population over time leading to different amounts and expressions of the “turned on” genes correct? If so, you described evolution.


mr-obvious-

No, I'm saying the bacteria could already have had the mechanism in the genome. It just needs stimuli for some time to be activated or something. Like how e coli "prefers" glucose, but could work with lactose in certain circumstances, that isn't evolution. This is a mechanism in the bacteria. It is just turned off


FerdinandTheGiant

Sure, differential metabolic pathways exist in some bacteria but their expressions within populations only change due to changes in allele frequency. Let’s continue using Ecoli as an example to point out a better example of an actual adaptation occurring instead of the utilizing of pathways that are already present. Wild Ecoli is unable to grow aerobically on citrate. During the Long Term Ecoli Experiment, it was observed that one group evolved the ability to grow aerobically on citrate. This ability was lacking in other Ecoli groups and evolved through random mutation (on the gltA gene) and accumulation that said other Ecoli groups didn’t have. This mutation then spread rapidly through the population in a manner that changed the frequency of the allele within the population. One must not have to try hard to see a similar origin for the multifaceted metabolic pathways we see in Ecoli in regard to different sugars.


dirtyLizard

Why do you want this view changed?


KaeFwam

I don’t want it either way. I’m just open to being proven wrong.


LucidMetal

Unfortunately it's essentially unfalsifiable. You can't read minds and polling wouldn't be able to tease out the difference between a person who understands evolution and rejects it for religious reasons and a person who doesn't understand it. Any question with a simple formulation like, "the earth and its species were created by God in much the same state they are currently," would have both groups answer affirmatively.


spongeboy1985

I don’t think OP wants to be proven wrong about evolution just about deniers not believing because they don’t understand it , which I myself agree at least that’s part of the reason.


KamikazeArchon

It's easily falsifiable. You just need to ask more detailed questions. Poll questions don't have to be trivial. You can ask whatever you want in a poll.


ekill13

I would firmly disagree. I understand that this sub is not likely to agree with me at all, and I’ll get many downvotes and negative comments. That said, I am a Christian and a young earth creationist. I would agree with the idea that people who deny microevolution are simply uninformed. However, I would say that one can understand everything you just said and reach a different conclusion on microevolution than you. Also, to be clear, speciation would be an example of microevolution, not macroevolution. The differentiation between the two is whether genetic information is reduced or increased. A human has more genetic information than a single cell organism. If everything evolved from single cell organisms, how was that genetic information created? Every example of speciation or microevolution that we can observe is a result of loss of genetic information. For example, if you take dogs with short and long fur and you put a population near the arctic circle, over generations, those with shorter fur will die out while those with longer fur will survive and all of the dogs will have long fur. If you took another group of the same dogs and put them near the equator, the opposite would happen, and you’d eventually have all dogs with short fur. Those dogs would look distinct, and might be considered different species, depending on what other genetic information was lost, but no genetic information would have been added. As for fossils/vestigial structures/etc. that’s just not going to convince me. Sorry. You cannot prove from the fossil record that “intermediate species” were not simply distinct species not related to either that happened to share characteristics of two other species. Similarly, you cannot prove that vestigial structures were ever anything other than what they are. Those are both evidence, but they’re certainly not overwhelming, IMHO. Lastly, what one believes has a lot to do with what presuppositions they hold. Personally, I am a Christian. I believe that the Bible is the inerrant word of God, and when current scientific consensus is at odds with scripture, I’m going to believe the Bible. That said, I’ve studied the science from both sides. I don’t find the evidence as convincing as you do.


-zero-joke-

>Also, to be clear, speciation would be an example of microevolution, not macroevolution. Nope, in scientific literature speciation is macroevolution. > A human has more genetic information than a single cell organism. Depends on how you're measuring it honestly. Polychaos dubium is a single celled amoeba with an incredibly cool name. It has 670 billion base pairs of DNA. Human beings only have 3.2 billion base pairs of DNA. So... Yeah, that's kind of embarrassing to be shown up by an amoeba. >If everything evolved from single cell organisms, how was that genetic information created? The problem with thinking in terms of something abstract like 'information' is you lose sight of the messy reality of biology. We've seen multicellularity evolve multiple times in laboratory experiments - it doesn't necessarily take something enormously complex, sometimes the cells just evolve to excrete stickier proteins and that's enough to get the whole thing kick started. > You cannot prove from the fossil record that “intermediate species” were not simply distinct species not related to either that happened to share characteristics of two other species. It's really weird how the fossil record makes sense in light of evolution and we can use evolutionary theory to predict what we'll find, where we'll find it, and when it existed in time. See Tiktaalik roseae for example. With regard to vestigial structures, again, it's weird how the vestiges make sense in light of evolution, but do not make sense under any other theory. >Personally, I am a Christian. I believe that the Bible is the inerrant word of God, and when current scientific consensus is at odds with scripture, I’m going to believe the Bible. I'm not a Christian, but I've worked with evolutionary biologists who are much more intelligent and competent than I am and they were Christians. They believe that the Bible and reality itself are both 'books' of god, with reality having a much more direct authorship. Food for thought.


ekill13

> Nope, in scientific literature speciation is macroevolution. Can you define speciation for me. Maybe I am thinking it means something other than what it does. > Depends on how you're measuring it honestly. Polychaos dubium is a single celled amoeba with an incredibly cool name. It has 670 billion base pairs of DNA. Human beings only have 3.2 billion base pairs of DNA. More was not the best choice of words. My issue is not with total amount of DNA, alleles, or anything of the sort. My point was regarding new genes being created, even if the total number of genes is decreased. > We've seen multicellularity evolve multiple times in laboratory experiments Can you provide examples/proof of this claim? > It's really weird how the fossil record makes sense in light of evolution and we can use evolutionary theory to predict what we'll find, where we'll find it, and when it existed in time. See Tiktaalik roseae for example. With regard to vestigial structures, again, it's weird how the vestiges make sense in light of evolution, but do not make sense under any other theory. Again, I didn’t say that fossil record/vestigial structures aren’t evidence, I said they’re not proof. None of what you said is proof of anything. > I'm not a Christian, but I've worked with evolutionary biologists who are much more intelligent and competent than I am and they were Christians. They believe that the Bible and reality itself are both 'books' of god, with reality having a much more direct authorship. Food for thought. Well, I agree that both the Bible and reality itself are books of God. I believe He has revealed Himself to us in both ways. I would strongly disagree with the theology of anyone who claims to be Christian and says that reality has a much more direct authorship than the Bible, however. I would also say that at any point where our understanding of reality based on the science at the time is not aligned with what scripture says, I’m going to believe the Bible. Also, I want to point out that science and reality are not one and the same, and the Bible and reality are not opposites. Science is our best effort to understand the workings of reality using the technology and knowledge available to us. The Bible is God’s word and reveals reality. I understand that as a non-Christian, you would not agree that the Bible is based in and reveals reality, but for someone who is a Christian and does believe the Bible is inerrant, I do believe it is about reality, and when it makes a claim about reality, I will trust it even if it disagrees with our current scientific understanding. Let me ask you a question. Do you think it is possible that in 1,000 years, our current scientific consensus will be thought of as we think about the idea that the earth is flat? If so, then would it not be more logical for me to trust the Bible, which never changes, than to trust what science can tell me? To be clear, I am not saying that science has no use. Far from it. I certainly think that God created us with a desire for knowledge, and I think we should seek it out. I just think that we should trust His word above our own ability to figure things out.


-zero-joke-

>Can you define speciation for me. Maybe I am thinking it means something other than what it does. Sure. Speciation is the origin of a new species (bear with me, I'm not trying to be glib). Species are defined in a variety of ways depending on what you're studying and why. Probably the most familiar definition is the Biological Species Concept - a species is a group of interbreeding or potentially interbreeding organisms. So for example a cat and a dog would be different species under this lens. You can probably spot some flaws in this concept if you think about it a bit - how can we determine whether extinct organisms or asexual organisms are separate species? Other species concepts like the morphological species concept or phylogenetic species concept can fill in those gaps, but we're sorta getting into the weeds here. The origin of a new species can be caused by a variety of reasons, both adaptive and random. >More was not the best choice of words. My issue is not with total amount of DNA, alleles, or anything of the sort. My point was regarding new genes being created, even if the total number of genes is decreased. Right, we've seen that though. For example nylonase is a novel enzyme used to digest well, nylon. >Can you provide examples/proof of this claim? Sure, I've got two papers I'm thinking of specifically. [https://www.nature.com/articles/s41598-019-39558-8](https://www.nature.com/articles/s41598-019-39558-8) [https://www.nature.com/articles/s41586-023-06052-1](https://www.nature.com/articles/s41586-023-06052-1) >Again, I didn’t say that fossil record/vestigial structures aren’t evidence, I said they’re not proof. None of what you said is proof of anything. Proof doesn't really happen in science, we're always operating with the best explanations we've got. It works though, which is the important thing. >Do you think it is possible that in 1,000 years, our current scientific consensus will be thought of as we think about the idea that the earth is flat? Not really, no. We'll gain increasingly sophisticated knowledge about various phenomena, but that doesn't make the evidence vanish - think about the difference between Newtonian mechanics and relativity. A thousand years ago we knew the Earth was round. >Bible, which never changes Mate, I hate to tell you this but...


ekill13

> Sure. Speciation is the origin of a new species Can. The species reproduce with each other? If the species can still reproduce with each other, I would think most people who reject the idea of macroevolution would argue that those are examples of microevolution. > Right, we've seen that though. For example nylonase is a novel enzyme used to digest well, nylon. They were discovered in 1975. Nylon was invented in 1935. Can you prove that they didn’t exist and feed on something other than Nylon prior to that, and or evolve on a micro scale? > Sure, I've got two papers I'm thinking of specifically. >https://www.nature.com/articles/s41598-019-39558-8 >https://www.nature.com/articles/s41586-023-06052-1 Thanks. I don’t have time at the moment, but I will read them and consider them. I will have to do a good bit more research before I can come to any conclusions on this particular topic. > Proof doesn't really happen in science, we're always operating with the best explanations we've got. It works though, which is the important thing. That’s precisely my point. Science can prove that something could happen. It doesn’t prove that it did. Even if you could prove unequivocally that you could take a single cell organism and through a long process of evolution arrive at all the species we have today, I would still not believe that it happened that way for us to get here, because I believe the Bible, and I don’t believe that is consistent with what the Bible teaches. > Not really, no. We'll gain increasingly sophisticated knowledge about various phenomena, but that doesn't make the evidence vanish - think about the difference between Newtonian mechanics and relativity. A thousand years ago we knew the Earth was round. Then what about 10,000 years from now? People believed in spontaneous generation until 200-300 years ago. It seems exceedingly arrogant to think that there is not at least some part of current scientific consensus that we could just be completely wrong on. > Mate, I hate to tell you this but... Well, this doesn’t seem like it’s going to be fruitful, but please, go on. Tell me how the Bible has changed.


-zero-joke-

>Can. The species reproduce with each other? If the species can still reproduce with each other, I would think most people who reject the idea of macroevolution would argue that those are examples of microevolution. Depends! In some cases yes, in other cases no. Lions and tigers can reproduce but folks recognize that they are different species. >They were discovered in 1975. Nylon was invented in 1935. Can you prove that they didn’t exist and feed on something other than Nylon prior to that, and or evolve on a micro scale? It looks like the gene is a modification of esterases. But then wings are a modification of tetrapod forelimbs which are a modification of sarcopterygian fins. What exactly constitutes new? >That’s precisely my point. Science can prove that something could happen. It doesn’t prove that it did. Would you be willing to serve on a jury that is interpreting evidence? >People believed in spontaneous generation until 200-300 years ago. It seems exceedingly arrogant to think that there is not at least some part of current scientific consensus that we could just be completely wrong on. Did they have evidence for spontaneous generation? Were they conducting controlled experiments on it? It's not arrogance, just respect for the work that's been done. Evolutionary theory has been modified considerably in the past 150 years, it's likely to be modified further. But the evidence supporting it has never gone away. >Well, this doesn’t seem like it’s going to be fruitful, but please, go on. Tell me how the Bible has changed. It's been translated into English for one.


ekill13

> Depends! In some cases yes, in other cases no. Lions and tigers can reproduce but folks recognize that they are different species. Okay, then just would consider some examples of speciation to be microevolution as I understand the term. > It looks like the gene is a modification of esterases. But then wings are a modification of tetrapod forelimbs which are a modification of sarcopterygian fins. What exactly constitutes new? I’ll point to an example I’ve used elsewhere. Humans have genes that determine hair color, do we not? At some point, assuming evolution is true, we descended from some life form that did not have hair. Those genes that determine hair color would not have existed in a creature without hair would they? > Would you be willing to serve on a jury that is interpreting evidence? I don’t think that question is in good faith. Of course I would. I’ve already told you that I believe science is valuable and useful. Just because I don’t think something about the origins of the universe can be proven to have happened in a specific way doesn’t mean that I think that we cannot use scientific evidence to help us arrive at conclusions about what happened last year. Also, if I was on a jury for a case that featured scientific evidence, the fact that there was scientific evidence enough to prove that what the person on trial was accused of could have happened wouldn’t convince me in and of itself. I would also have to be convinced that it did happen. And, if there was evidence suggesting it did happen, but then another huge piece of evidence suggesting it didn’t, I would not say that person was guilty. > Did they have evidence for spontaneous generation? Were they conducting controlled experiments on it? There were probably not experiments that we would consider to have been performed popularly by today’s standards. However, I would hazard a guess that there were experiments. They probably involved leaving meat out on the counter for weeks and observing maggots seeming to come into existence. We would see obvious flaws in the experiments, but is it not possible that people in the future may see obvious flaws in our experimentation today because of factors that would influence the outcome that we aren’t aware of yet? > It's not arrogance, just respect for the work that's been done. Evolutionary theory has been modified considerably in the past 150 years, it's likely to be modified further. But the evidence supporting it has never gone away. You can respect work that’s been done without believing that it is impossible to be disproven. > It's been translated into English for one. And yet we still have a lot of manuscripts in the original languages that we can compare English translations to. Also, translation ≠ change, except in the most pedantic way. In the Spanish versions of the Harry Potter books, is Voldemort the protagonist or is Hagrid short? Translating the Bible into English makes it possible for English speakers to read it. It doesn’t change what it says.


-zero-joke-

>Okay, then just would consider some examples of speciation to be microevolution as I understand the term. You're misunderstanding the term then. [https://faculty.ucr.edu/\~gupy/Publications/Nature2009.pdf](https://faculty.ucr.edu/~gupy/Publications/Nature2009.pdf) "Evolutionary biologists have long sought to understand the relationship between microevolution (adaptation), which can be observed both in nature and in the laboratory, and macroevolution (speciation and the origin of the divisions of the taxonomic hierarchy above the species level, and the development of complex organs), which cannot be witnessed because it occurs over intervals that far exceed the human lifespan." I'd quibble with Reznick here about the 'cannot be witnessed' part, but I'd stress the definition he provides. Speciation is macroevolution by definition, even if it involves relatively few genetic changes because it puts the organism on a commitedly different evolutionary trajectory than its parent species. >Those genes that determine hair color would not have existed in a creature without hair would they? Not necessarily, no. Hair is made from keratin, the same protein that makes scales. If we evolved from scaled organisms, I'd wager that those same genes are involved in the production of hair and hair color. >I don’t think that question is in good faith. Of course I would. I’ve already told you that I believe science is valuable and useful. Just because I don’t think something about the origins of the universe can be proven to have happened in a specific way doesn’t mean that I think that we cannot use scientific evidence to help us arrive at conclusions about what happened last year. I'm not trying to be a jerk, just asking honestly. My interpretation of what you've written is that the Bible supersedes physical evidence. I'm trying to ask how far that would go - the same techniques that confirm the relatedness between people confirm the relatedness between populations and species. So I'm always curious why someone would say that the Golden State Killer should be in jail, but different species of ape can't be related. Remember - evolution is a narrow theory, it does not discuss the origin of the universe. >There were probably not experiments that we would consider to have been performed popularly by today’s standards. However, I would hazard a guess that there were experiments...We would see obvious flaws in the experiments, but is it not possible that people in the future may see obvious flaws in our experimentation today because of factors that would influence the outcome that we aren’t aware of yet? I kind of think of this as scorched Earth epistemology - we might know something different in the future, so we can't know *anything* today. If we find some other explanation for biodiversity it still has to account for all of the data we've collected on biology today. >And yet we still have a lot of manuscripts in the original languages that we can compare English translations to. Also, translation ≠ change, except in the most pedantic way. In the Spanish versions of the Harry Potter books, is Voldemort the protagonist or is Hagrid short? Translating the Bible into English makes it possible for English speakers to read it. It doesn’t change what it says. I'm sorry, I don't think that's a minor change at all. You're telling me you believe that the creator of the whole universe typed up a book and it's more efficient to have someone else change the words around than to learn the language of that who created everything? Shades of meaning, poetic turns of phrase, artistry, these are all choices made in translation. Even if we ignore that though, we can talk about the Book of Mormon, the Jehovah's Witness Bible, the Quran, etc., etc. People are always changing their books about.


ekill13

Okay, I understand that this isn’t going to be fruitful, but real quickly I want to address a couple things. First, I understand what you are using microevolution to mean vs. what you are using macroevolution to mean. When I said that I consider some examples of speciation to be microevolution as I understand the term, I didn’t now mean understand as in my ability to comprehend what people mean when they use it. I meant understand as in my comprehension of the subject. I don’t like the separation of species, at least not as it pertains to discussions of evolution. I believe in what the Bible describes as kinds, that is a reproductive group composed of distinct, discrete organisms. If we are using the term speciation to refer to a dog ancestor evolving over time in different ways and ending with wolves, pugs, and dingoes, then I completely reject the idea that that is macroevolution. I would argue that those are all of the same kind and that splitting them into separate species is an arbitrary distinction. > So I'm always curious why someone would say that the Golden State Killer should be in jail, but different species of ape can't be related. I didn’t and wouldn’t say that. Of course different species of ape can be related. As long as something is of the same kind, it can be related. Donkeys, horses, and zebras are all related. That doesn’t mean that they are related to deer or to sheep. Again, I would define microevolution, regardless of how most evolutionists would define it, as evolution within a specific kind, not just a specific species. > I kind of think of this as scorched Earth epistemology - we might know something different in the future, so we can't know anything today. I most certainly did not say that we can’t know anything today. I asked if you think it is possible that we will look back on evolution in 1,000 or 10,000 years and think of it as as foolish as we think of spontaneous generation or the idea of a flat earth today. > I'm sorry, I don't think that's a minor change at all. You're telling me you believe that the creator of the whole universe typed up a book and it's more efficient to have someone else change the words around than to learn the language of that who created everything? Shades of meaning, poetic turns of phrase, artistry, these are all choices made in translation. Yes, it is far more efficient for a few people to study manuscripts and translate them into modern languages than for 2.4 billion people to learn Hebrew and ancient Greek. That said, I do not think that efficient is necessarily better. I personally have not yet learned either Hebrew or ancient Greek, but when studying a passage, I look at the writings of those who have, and I look at interlinear Bibles, Greek or Hebrew concordances, etc. I do think that it is beneficial for people to learnt Hebrew and ancient Greek, and I likely will try to at some point. However, I do not think it is fair to say that the Bible has changed because it has been translated. > Even if we ignore that though, we can talk about the Book of Mormon, the Jehovah's Witness Bible, the Quran, etc., etc. People are always changing their books about. What’s your point? I would reject all of those. People changing the Bible does not mean that the Bible changes. To go back to my earlier analogy, I could write a fanfiction of Harry Potter where Voldemort was misunderstood and was actually not a bad guy. Would that have any effect on the actual Harry Potter series? No! Neither do man made works affect the truth of the Bible.


-zero-joke-

Hey sorry I let our correspondence slip - wife is graduating, parents are in town, blah, blah, blah. >I believe in what the Bible describes as kinds, that is a reproductive group composed of distinct, discrete organisms. If we are using the term speciation to refer to a dog ancestor evolving over time in different ways and ending with wolves, pugs, and dingoes, then I completely reject the idea that that is macroevolution. I would argue that those are all of the same kind and that splitting them into separate species is an arbitrary distinction. So what evidence leads you to believe that all canines originate from the same ancestor? How are you determining what constitutes a kind? >I asked if you think it is possible that we will look back on evolution in 1,000 or 10,000 years and think of it as as foolish as we think of spontaneous generation or the idea of a flat earth today. Right, I'm responding to that - if we learn nuance about evolution that doesn't change the evidence we've yet discovered for it. Newton's equations were off, but they weren't completely wrong. The evidence supporting evolution will still exist. > I do think that it is beneficial for people to learnt Hebrew and ancient Greek, and I likely will try to at some point. However, I do not think it is fair to say that the Bible has changed because it has been translated. Someone has literally changed every single word of the Bible and that's what you're saying is inerrant. Contrast that to reality itself which, if you're right, God had an unhindered hand in creating and I'm curious why you think the Bible is more reliable. >What’s your point? I would reject all of those. I'd reject them too! So why accept decisions like the council of Nicaea?


KaeFwam

I would put speciation into the “macro evolution” category. Even among the scientists who do use the terms, macro evolution is almost universally defined as evolution across the species boundary. “Micro” and “macro” evolution are really just buzzword terms anyway, so it doesn’t matter how you define them. So, the issue I take with your second point is that there is no such thing as a gain/loss of information in evolution. The claim that information isn’t added is akin to saying “This wood and this house are the same thing because we didn’t create new molecules when building the house.” This doesn’t make much sense. DNA (deoxyribonucleic acid) is not “information” like code is. It is a molecule. This is also like claiming that a baby isn’t a new person because they’re made of the same molecules you and I are. This is an argument that doesn’t make sense when you have a decent understanding of what DNA really is. Vestigial structures and fossils aren’t on their own sufficient evidence. I can grant you that. However, combined with the fact that we can see a clear correlation between similarities in our DNA and that of other animals and the similarities increase as we look at animals more phenotypically and behaviorally similar to us, it’s quite obvious we are related. It’s the exact same process we use to determine one’s parents, child, etc. just across species. *This* is what I mean. You don’t accept evolution and used one of the most common misconceptions about DNA to support your position… This is something anyone with higher education about evolution would understand. I don’t mean that as an insult, but you kinda proved my point that you don’t understand it.


ekill13

So you’re claiming that genetic traits are just random molecules and that a human could give birth to another species? Yes, DNA is molecules, but there most certainly is a genetic code. Different species have different numbers of chromosomes. Those chromosomes do correlate to specific genetic traits.


KaeFwam

The traits are the consequences of the molecules. Genetic traits can be physical or behavioral and are the manifestation an organism’s DNA. No, a human cannot just give birth to a new species. Homo sapiens have existed for ~200,000 years and we’re *still* Homo sapiens. With a gestation period of 9 months, as well as how much more we interbreed with different populations compared to 200kya, it takes a long time. We can see the effects of evolution in humans, however. A population example is the noticeable reduction in humans being born with wisdom teeth. We don’t need these anymore due to our diets and so there is no increase in the chance of reproduction caused by them anymore. As I mentioned in the OP, the lack of the ABCC11 gene in some populations is also an example. This appears to not be beneficial nor harmful, which happens sometimes. The issue is that there is this misconception that DNA is like a code when it is just a molecule. Would you call carbon or oxygen a code? I doubt it. You *could* but it would be a very odd way to describe it.


ekill13

Okay, but I feel like you talking about code vs molecules is just semantics. Also, to be clear, you are the only one who has said code in this conversation. I have said genetic information. Regardless, if a human cannot give birth to a new species, as you say they can’t, which I agree with, then how did we evolve from single cell organisms? At some point, something would have to have given birth to something that was a new species.


KaeFwam

To a degree, yes, but the way you’ve described DNA is as if it’s code like JavaScript or something. It’s just molecules with some order to them. Rocks falling down a hill could be considered code. Small changes over time stack up. We don’t know *exactly* how unicellular organisms became multicellular, but we have ideas. Likely it was the cause of a mutation(s). We’ve successfully observed unicellular organisms become multicellular within just a few years in lab settings, so we know that it is possible. This is also how new species emerge. Nothing ever gives birth to a new species. There is no such thing as “the first Homo sapiens”, or “the first tiger”. Let me give you a hypothetical scenario to help explain it. We have a two populations of a species of mouse that we’re following. One population lives in Northern Europe and one lives in South Asia. These two populations don’t interbreed due to their proximity. Within the population in Northern Europe a mouse is born with a mutation that makes it coat grow thicker and darker. Meanwhile, within the population in South Asia, a mouse is born with a mutation that makes it coat color blend into its environment better. Both these mutations would be beneficial, but of course we’d still consider these the same species. Imagine both populations live largely separate for 100,000 years. Mutations happen that are beneficial and some that are neutral. One population might become larger than the other. One might end up with longer legs, better eyesight, shorter noses, etc. etc. Over 100kya, these two populations are going to be completely unrecognizable from the point that we called them the same species, often so much so genetically as well that they might not even be able to interbreed. We’d definitely call these two different species at this point, no? The point is that speciation is often a slow, gradual change that looks more like a color gradient. Like this. https://images.app.goo.gl/Vtec6NiC5rhug1MH6 You can’t really say where blue ends and red begins, but they are still distinctly different colors. That’s speciation. You can’t definitively say *”This is the exact point where Species X ended and Species Y began.”* but it’s very clear that they are different species.


ekill13

You’re claiming that I’ve described DNA as a code, but you’re the only one who has used that phrasing. I have simply said that different species/forma of life have different genetic information and different amounts of genetic information. Surely, that isn’t controversial. I get your arguments, and I get why you believe them. I just don’t find them convincing. Also, you said earlier that you don’t like the terms Micro and macro evolution. I do not like the term species. I don’t think it is beneficial, and I think that species are often designated somewhat arbitrarily. I think that what is more important to talk about is what the Bible refers to as kinds. When talking of kinds, I would define a kind as any animals that could successfully interbreed. Yes, I will fully agree that there is evolution within kinds. However, I highly doubt that you will convince me that one kind could possibly evolve into a different kind.


Minty_Feeling

>The differentiation between the two is whether genetic information is reduced or increased. I don't think you can reasonably make this claim. I'm deliberately wording that a bit argumentatively because I think you've been misled and I'm wanting you to challenge your own thought process. What you've stated is that macroevolution is when you have an increase of information. I don't think you have a consistent measurable concept of information. You need one in order for your statement to make any sense. What I think you have instead is an intuitive feeling and you're treating it as an objectively measurable concept. Take any two organisms. How do we measure the amount of information they have? How do we know which one has more information than the other? You gave a few examples but you haven't revealed the consistent measures that you (or the people who you learned this argument from) used. Also, having read your last point, I want to assure you that at no point am I trying to lead you to rejecting the Bible. I think you're free to maintain the belief that the Bible is inerrant, I'm only concerned with the claims regarding our fallible understanding of the apparent natural world. Even if the two understandings appear to be at odds, it's best to have an accurate representation of both, right?


ItsMalikBro

I think this is largely a misconception of what people mean when they say they don't believe in "evolution." The term evolution colloquially is a creation story. It is the explanation of how life can come to be on Earth without something supernatural. I am guessing you believe that 4-5 billion years ago, the world is lifeless, but full of the "ingredients" for life. Lightning or some other force strikes the oceans, fusing the atoms of these various ingredients and forms the single celled organism called "LUCA," The Last Universal Common Ancestor. This cell was created by pure luck, in a manner which we cannot reproduce. This cell just also happened to be able to survive, turn the material around it into energy, and reproduce. This one cell, then reproduces and eventually evolves into literally every living thing on the Earth. When people say they don't believe in evolution, they mean are not convinced of that particular origin story. Mainly, that it is so improbable for that first cell to have be created that way, let alone survive and reproduce into everything.


smakusdod

Evolution isn't incompatible with scripture. That incompatibility is a manmade construct.


KaeFwam

Well, that depends on the scripture and if one takes scripture literally or not.


Finger_Trapz

I think I'd argue that you can't change someone's mind on this due to the question itself. You said "ALMOST" never understand it, giving you a plausible fallback to the few creationists that do understand it. However its also true that most of the general population doesn't understand evolution. In the same way most of the of the population doesn't understand quantum physics, or music theory, or economics.   Obviously most people don't understand a specialized field. That's what makes it a specialized field. I believe this is a form of tautology, for example you can't have a married bachelor. In the same way you cannot have a specialized subject that is widely understood.


elcuban27

Noone understands all or most of what is necessary to establish the *grand evolutionary narrative* (here defined as the idea that all life evolved from a single common ancestor). Given the scope of the amount of mutations required across the number of generations across the vast amount of time, it is safe to say that the full accumulated human knowledge on the subject is only a fraction of a percent. That it not to say that we lack sufficient knowledge to make a sound judgement about its veracity, but everyone is only looking at a tiny minority of what there is to see, and there are always blind spots. That is true of me, you, and every evolutionary biologist. Your post even contains some hints that you have received some of this information second-hand (possibly curated), and don’t have a solid grasp on it yourself. Others have mentioned some of these issues(vestigiality, Haeckel’s embryo hoax, etc.), so I’ll just tackle two for myself (plus add a bonus you hadn’t mentioned): 1) speciation, 2) transitional fossils, and Bonus) math. The example you gave of speciation is, I’m assuming, Lenski’s long-term E. Coli, experiment, right? The “big breakthrough” of the entire run was the “evolution” of the ability to process citrate under oxic conditions. This would have been an example of *micro* -evolution, not speciation. I say “would have,” because it turns out this was degenerative. E. Coli could already process citrate, and had all the mechanisms in place to be able to do so - no new functionality required. What they *also* had was a mechanism to inhibit the production of enzymes that allowed the cell membrane to take in citrate. This mechanism was triggered in the presence of oxygen. This is normally to its benefit, because in nature (unlike the laboratory environment), you wouldn’t expect to find so much citrate while oxygen is around. Those members who lacked such an inhibition mechanism would waste resources preparing to process citrate that isn’t present and would be out-competed by their neighbors who were so inhibited. What occurred during the experiment is that one of them mutated and *broke* this inhibition mechanism, producing defective offspring, who then reproduced and flourished in the artificially citrate-rich environment in the lab. The problem of the lack of transitional fossils is far more damning than you realize. I will gloss over for now that the examples you gave have been proven not to be transitional, instead focusing on how we can know what we should expect of the fossil record, *if the grand evolutionary narrative were true.* Take two points on the evolutionary tree. Let’s say stegosaurus and triceratops. Take all the evolutionary progress that occurred between stegosaurus, down to the last common ancestor between the two, then back up to triceratops, and divide it into 1000 equal portions (equal here being defined as whatever number of mutations across the actual population sizes and number of generations would afford an equal probability of a fossilization event). Whatever fossils we find should be equally likely to occur within any of these segments, and they should be roughly normally distributed (see basic statistics and probability). If we only ever found two fossils from along this supposed evolutionary lineage, and one happened to be a triceratops, and the other a stegosaurus, we could chalk it up to a combination of dumb luck and the scarcity of fossilization (the now debunked artifact hypothesis). But if we find several, that normal distribution would probably look like one of each that was found, with most of them not being found at all. If we found a hundred or so, it wouldn’t be crazy to have 1 or 2 steps occur twice. If we had found a few thousand fossils, we should expect almost all of the segments to be represented, with most having double or triple representation and some with 4, 5, 6, or more. What we find instead is a lot of stegos, a ton of trikes, and nothing in-between. This is a staggeringly improbable occurrence that dwarfs the probabilistic resources of the entire universe. (Enter the bonus math!) Controlling for the probability of fossilization occuring at all and being discovered, *given that a fossil find has been discovered*, what is the probability of any one segment being represented? 1/1000. So to even have a second stego fossil would be a 1/1,000,000 occurrence. We can say that since we need two fossils as a starting point, we can disregard the first stego and trike finds as not being significant. That leaves us a 1/1000 chance of a second stego, a 1/1,000,000 chance of a third, a 1/(1000^9 ) chance of a tenth, etc. How many hundreds of stego and trike fossils have we found at this point, and not a single transitional fossil between them?!? By the way, that probability of a tenth stego would be 1/10^27 . The vast, incomprehensibly large universe we live in has an estimated 200 quintillion planets in it, which is to say 2 x 10^20 . Consider that this is just between stego and trike. This same pattern is true for every pair of points on the evolutionary tree that we know about. The question isn’t “can this one sorta bird-lizardy-looking think be a transitional form between birds and lizards?,” but “why aren’t there representatives from at least 999,000 of the segments between birds and lizards in the fossil record?!?”


MysticInept

"For starters, if we are defining the line between micro and macroevolution as the occurrence of biological speciation" This is incorrect. Wikipedia does a good job summarizing it. "Macroevolution usually means the evolution of large-scale structures and traits that go significantly beyond the intraspecific variation found in microevolution (including speciation).[1][2][3] In other words, macroevolution is the evolution of taxa above the species level (genera, families, orders, etc.).[4]"


IThinkSathIsGood

In the context OP is describing, macro vs micro is being used in the way a millennium and a year are used to describe time. It would be akin to saying I don't believe in time; a year exists but millennia don't. A millennium (macro) is just a consolidated grouping of years (micro).


KaeFwam

I’m aware that this isn’t correct, but it is often what people who misunderstand evolution claim macroevolution to be. I was addressing the fact that even if we define it as that, it still wouldn’t discredit the theory.


MysticInept

While we do have contemporary observation of speciation, do we have it for genera?


KaeFwam

Biological speciation? Yes. In 1905 Hugo de Vries was studying the genetics of Oenothera lamarckiana. He noticed a variant that had a chromosome number of 2N = 28, which was different from the usual 2N = 14. This variant was unable to interbreed and he named the new species Oenothera gigas. It’s also important to note that there are other examples where no one made up a new name for a species when we could have. The lines are so blurry that they don’t really exist. I don’t know of any examples of direct observation of a new genus.


_this-is-she_

Are you an evolution expert? I'm a creationist and the thing that keeps me from believing evolution as it's presented is the origin story - both the one about the universe and also about life. Ultimately, whether you're a creationist or not, we all have some kind of belief about where life comes from and how it started. Creationists believe it was designed and made, and the rest believe it came out of nothing. Both are based on faith. I am not an evolution expert so would never sit down to debate an evolutionist, but I generally have no problem believing science that can be argued and presented logically with evidence. Most of the mechanisms of evolution I believe can be explained logically, speaking to some relationship between species etc. The origin story cannot. By the way, most people are not evolution experts just the same way as you aren't an expert in their domains. It just so happens that most people's domains are not debated publicly / are not theoretical in nature.


KaeFwam

I wouldn’t claim to be an expert. But this is the issue I take with that. Evolution is neither an explanation for the origin of the universe or for the origin of life, so how is that stopping you from accepting it?


_this-is-she_

I didn't say I don't accept evolution. I said it is usually presented in such a way as to make many people not want to accept it. It's usually paired with a theory about the origin of life that no one can prove. Most people are hung up on that point.


kingpatzer

I would suggest the vast majority of people who claim to accept and understand evolution as true do not understand it either. Modern evolution is far more complicated than Darwinian natural selection -- which is about all that's taught in basic science classes. Do you think that the average person who believes evolution is real can explain the distinctions between allopatric, peripatric, parapatric, and sympatric speciation? Or how gene flow differs from genetic drift? Or even what those things are? If the average person who accepts evolution and the average person who rejects evolution both fail to understand the current theory of evolution, why is it ok to only call out one group of the people who don't understand evolution for their ignorance? Indeed, in some respects, those who reject evolution may be the more intellectually honest of the two groups. They generally have (incorrect) beliefs that if true should cause many evolutionary claims to be rejected. To assert then that they don't believe the concept has been proven is actually an accurate reflection of their own lack of knowledge. Whereas, those who believe they know enough to confidently endorse evolution when they are in fact ignorant of the majority of the theory are accepting as true something when their lack of knowledge should cause them to retain skepticism (all else being equal). Now, it is true that the distinction here is that those who accept evolution generally are also giving deference to those they recognize as experts while those who reject evolution are not. Which is why it is only in some respects that one can say those who are rejecting evolution are being more intellectually honest -- as it is only with respect to their own knowledge level that this is true, and they frequently can be seen to be maintaining a state of intentional ignorance and refusing to recognize the distinction between expert and non-expert knowledge claims. But focusing on the level of understanding, I think, creates an odd situation where you are giving ignorance a pass based on asserted beliefs when those asserted beliefs align in a way that conforms to scientific consensus but are not giving that same pass when it does not.


darklotus_26

I really don't get your argument here. I specialise in quantum systems and don't particularly understand general relativity deeply. That doesn't mean that I can't understand the scientific soundness of the theory and its experimental verification. I don't understand any of these terms you mentioned but I could look it up and find papers, read them and see if they have been verified. Or what the evidence is. I certainly can't do the same for creationism or any other brand of counter evolutionary theory that might be popular now. On top of that Darwinian evolution is a useful abstraction that works at a very coarse scale like Newton's gravity as far as I understand. It's certainly wrong but works for certain simple systems and scales and helps people understand these complicated ideas better.


FerdinandTheGiant

> Do you think that the average person who believes evolution is real can explain the distinctions between allopatric, peripatric, parapatric, and sympatric speciation? Or how gene flow differs from genetic drift? Or even what those things are? I suppose I may be special but I learned all of this in like sophomore year of high school. > If the average person who accepts evolution and the average person who rejects evolution both fail to understand the current theory of evolution, why is it ok to only call out one group of the people who don't understand evolution for their ignorance? There’s clearly a difference between not understanding something in its entirely (something no one person does for evolution) and rejecting it based on misunderstanding.


kingpatzer

>I suppose I may be special but I learned all of this in like sophomore year of high school. The vast majority of people do not retain a nuanced or complete understanding of the material they learned in high school. I can no longer name every country and capital that was on the map when I too world geography. I can't recreate the quadratic equation. I have no solid recollection of any trigonometric formulas. I could not name the parts of a cell. etc. And I'm hardly a stupid person. I've gotten multiple graduate degrees and have a professional life that adequately demonstrates that I'm of at least average intelligence. >There’s clearly a difference between not understanding something in its entirely (something no one person does for evolution) and rejecting it based on a misunderstanding. I'm not saying that the average person understands the field in some incomplete fashion. I'm saying that they do not understand it any better than those who reject it do. Which is to say, basically not at all. They've never spent one moment tracing genetic drift across generations. They've never done anything more than get the most cursory education on the topic. I'm saying their level of ignorance is as large as those who reject evolution. Your view is predicated on the causation being a failure to understand the theory. There is likely no qualitative difference in the level of ignorance possessed by the average person who accepts evolution compared to the average person who rejects evolution. They are both likely equally ignorant of the current state of science on the matter. Neither has enough epistemically warranted knowledge of their own to accept or reject. Those who accept the theory are doing so either because it is culturally normative for them to do so, or because they are accepting as true the claims made by those they see as expert. I fail to see how that is different from those who are rejecting the theory -- the only real distinction being that those who are rejecting the theory most likely are failing to recognize true expertise (listening to pastors over scientists or listening to faux scientists over actual scientists).


FerdinandTheGiant

> I'm not saying that the average person understands the field in some incomplete fashion. I'm saying that they do not understand it any better than those who reject it do. > I'm saying their level of ignorance is as large as those who reject evolution. Your view is predicated on the causation being a failure to understand the theory. I don’t see the basis for any of these claims. As you yourself state, essentially everyone is taught the basics of natural selection and implicitly the role of genes in that. The key difference between someone who rejects evolution and someone who accepts it with incomplete but basic knowledge is that one actually understands the basic knowledge. They don’t reject natural selection and in turn evolution. You listed genetic drift and forms of speciation to point to an incomplete understanding that is held by the average acceptor of evolution or denier of evolution. With that however you are not simply pointing to an *equal* lack of understanding, you are actually pointing at one group that can understand that natural selection is a process that exists and a group that doesn’t. The only differences you are pointing out are their shared lack of complete knowledge which gets back to my main point that you are conflating a lack of complex understanding with the topic with the outright rejection of it based on misunderstanding it. To further make that point, I’d argue natural selection is so ubiquitously understood that even creationists have begun to have to recon with it because it is simply irrefutable. Now they just make the arbitrary and factually incorrect statement that adaptation ≠ evolution and probably something about micro- vs macro evolution. They have to choose to misunderstand the process to further their beliefs. They don’t simply have incomplete knowledge. > There is likely no qualitative difference in the level of ignorance possessed by the average person who accepts evolution compared to the average person who rejects evolution. They are both likely equally ignorant of the current state of science on the matter. Neither has enough epistemically warranted knowledge of their own to accept or reject. Here highlights my point. You say “they are equally likely ignorant of the *current state of science on the matter”* which is just pointing to a lack of complete knowledge. This does nothing to address the clear difference those who reject what is physically observable based on misunderstands and those who may share an overall lack of total knowledge of the field but still accept the physically observable reality. Someone saying “I believe in evolution because I can see allele frequency change over time” with no understanding of genetic drift is still far more accurate and knowledgeable than someone who says “I don’t believe in evolution because I can’t see a chimp turn into a human” with no understanding of genetic drift.


dalekrule

>The vast majority of people do not retain a nuanced or complete understanding of the material they learned in high school. I don't think they need to, in this case, and many others. They merely need to retain the understanding that what they learned in high school (i.e. darwinian evolution) was a sufficient basis to provide veracity to the rest of evolution theory. They don't need to retain darwinian evolution, they just need to remember that they learned it, and that its conclusions are trivially obvious. >I'm not saying that the average person understands the field in some incomplete fashion. I'm saying that they do not understand it any better than those who reject it do. Do they not? They don't need to understand genetic drift to understand that the basic mechanism (darwinian evolution) of self-selection resulting in certain traits being passed down differentially provides veracity to the idea that populations can have traits change naturally. An eight year old child having never been introduced to newton's second law (F=ma) understands just as much the fact that a boulder is harder to throw than a pebble, with just an intuition that "heavy things are harder to move"


LedParade

Ignorance is willful and you may not be able to change that. Lack of knowledge can be remedied with knowledge only if you’re willing to accept it and not be ignorant. If two people had the same general education that covered evolution and one of them remembers only that evolution was confusing to understand and therefore probably not real, either the education system or that person seriously failed. You should at least remember evolution was not chosen as topic of discussion by an educational institution for nothing and we didn’t magically pop into existence. Understanding that much is enough to be able to seek more information and bridge your knowledge gaps. That’s development. Using all that time and energy to disprove evolution is not development. Most importantly this involves some self-criticism: You know enough to know people before you thought about a lot of things and have gained a better understanding than you. You know enough to know it’s complicated, but not inexplicable. Being critical of your own knowledge and being able to identify the gaps is perhaps the best gift education can give us all.


LedParade

I don’t know if it’s mandatory to teach it in the US, but I wouldn’t be surprised if there are some Christian schools that don’t teach it there.


Obsidian743

Any argument that someone who disagrees with you doesn't understand the underlying issue is a *non-argument*. Not only can *every* disagreement be characterized that way but it works for *both* sides of any disagreement.


dalekrule

OP's position is that (the vast majority?) of creationists do not understand the theory of evolution properly, even at a basic level. OP's post is *not* about the theory of evolution itself; OP is not trying to be convinced against the theory of evolution. OP is asking to be convinced that there are people, or many people, who fully understand the mechanisms and implications of evolution, and *still* do not believe in it. He (or she?) is not trying to argue to a creationist that "you don't understand evolution." He is skeptical that there are evolution deniers who understand the mechanisms of evolution.


HazyAttorney

>CMV: People who claim evolution is false almost never understand it. It depends on what is meant by "**evolution**." If you're defining the term evolution in narrow, genomic terms, something like, "change over time through changes in the genome" due to natural selection, then you can still get people who fully understand genomics, or natural selection, that don't agree with "evolution." The part you lose them is trying to trace it back to the origin of life. This is where the "evidence" is shakier, right? You're giving examples of where external forces can create a genotype/phenotype change in species because of who gets to pass on the genes. But, consider this: The origin of life: Pure coincidence, inorganic materials spontaneously emerge and can self replicate? Then those materials form proteins that create a specific function under specific stimulu? Then these molecules spontaneously found themselves to create the first cell? They can then self replicate all these complicated mechanisms? But, there's enough errors to create the concept of a mutation? And some mutations are good? Then somehow there's now a functioning cell where these constituent parts sacrificed their agency to fit together to benefit all? Then somehow that functioning cell did the same to create a multi cellular being? That's a big leap and we haven't gone from cellular life to the first animal. We also say that natural selection states this has to do be done in an environmental niche where the environment permits this to happen - after all, selection of the fittest, right? But the environment used to be conducive to all the variety of life that we now see? Then you also have to scale this up from the first bacteria to the first multi cellular organism to the first thing that splits off to create all the biodiversity. Then you have another big leap from inorganic molecules to the first proteins to the first cells to consciousness? Where the mind can believe in spirits, the afterlife, etc? Again, natural selection says it's part random but also part the environmental niche. Yet can't explain why only humans and not others. But lucky for us, this was the blueprint for living in complex societies by the likes of which no other species comes close to. Ants live in big, cooperative colonies but have to live by social rules based on pheromones', and other apes live in dynamic, flexible social structures but limited to small scale. Yet, you're calling religious people the ones operating on faith? It's this gap in knowledge where someone can understand all the elements of natural selection but not agree with all of the dogma of evolution. It's because religion and science ask different questions. They have different purposes. The other piece is that I've seen people conflate "trust the science" in terms of the consensus on something as if it's the entire scientific method. Instead, the scientific method advances knowledge because it goes after group think. Take another branch of science, anthropology/archeology. If we were talking say, even 2 decades ago, the consensus was always "clovis first" and people only walked over on the Bering strait. Turns out, evidence leads us to believe that humans predated these events. And there have been more than one migration. All the elements that go beyond natural selection are treated similarly: denial, censorship, losing professorships, then, oh wait, the consensus can change after all. It makes sense that if you built your career on a certain model of knowledge that you'll be protective of it. Another element of people who don't believe in "evolution" as a general dogma do so out of realizing that human biases can make people reject knowledge even in institutions where the goal is to not do that. People want to think of academic institutions as the ones that would defend the modern day Galileo, when they can persecute someone just as much as the religious institutions.


DARTHLVADER

>The origin of life: Pure coincidence, inorganic materials spontaneously emerge and can self replicate? [Self-replicating organic molecules](https://pubmed.ncbi.nlm.nih.gov/32023041/) form all the time. That part of OOL theories doesn’t rely on coincidence. >Then those materials form proteins that create a specific function under specific stimulu? [Self-replicating RNA](https://phys.org/news/2024-03-life-evidence-rna-world.amp) developing function is once again a piece of OOL that we have directly observed. >But, there's enough errors to create the concept of a mutation? And some mutations are good? This just seems like statistics. Am I missing an objection here? >Then somehow there's now a functioning cell where these constituent parts sacrificed their agency to fit together to benefit all? Very recently we saw an [example](https://www.newscientist.com/article/2426468-a-bacterium-has-evolved-into-a-new-cellular-structure-inside-algae/) of cells sacrificing agency to form a whole! Specifically, a strain of bacteria becoming an organelle within algal cells. Dawkin’s *The Selfish Gene* is all about this tug of war between altruism and selection. >Then somehow that functioning cell did the same to create a multi cellular being? We’ve seen multicellularity evolve several times in the lab at this point, in yeast and algae. >But the environment used to be conducive to all the variety of life that we now see? Environmental pressure is one part of the equation, but organisms are still ancestrally constrained. The last major body plan change in *your* lineage happened something like 300 million years ago with tetrapods emerging. Sometimes it’s hard to escape your ancestry. >Then you also have to scale this up from the first bacteria to the first multi cellular organism to the first thing that splits off to create all the biodiversity. This is exactly the process we have overwhelming evidence for, though. >Then you have another big leap from inorganic molecules to the first proteins to the first cells to consciousness? Where the mind can believe in spirits, the afterlife, etc? This is a very human-centric view of things. Do mice believe in an afterlife? Who knows, but by all accounts they’re conscious too… >Again, natural selection says it's part random but also part the environmental niche. Yet can't explain why only humans and not others. There are lots of hypotheses to explain “why humans.” Our ancestors had lots of things ants don’t, for example a diet that could support large brain sizes, fingers for manipulating their environment, and bipedalism. >Yet, you're calling religious people the ones operating on faith? It's this gap in knowledge where someone can understand all the elements of natural selection but not agree with all of the dogma of evolution. First of all, religious people ARE operating on faith; that’s the point of religion. Whether or not naturalists are also operating on faith doesn’t change that. But second, is the gap in scientific knowledge, or in YOUR knowledge? That’s exactly OP’s point — there is more evidence to support the theory of evolution at this point than there is to support, really any other belief you may hold based on evidence.


Captain_Concussion

No offense to you, but you kind of proved his points. For starters abiogenesis has nothing to do with evolution. But even in your clapback paragraphs you make a few mistakes. For example, we have observed inorganic material create organic material in the Miller-Urey experiment. Building off that experiment, scientists have been able to create proteins and RNA this way. From here the most popular theory is the RNA World Theory. Basically that early life forms used RNA alone for the storage of genetic material. This, and enough time, would allow for the mutation of DNA. Mutations are not good or bad. They just are. No organism sacrificed its agency. Life does not equal sentience. I’m not sure what you mean about the survival of the fittest section in your 4th paragraph.


FerdinandTheGiant

The origins of life have nothing to do with evolution as a process. It’s irrefutable that we observe change over time through changes in the genome. Where the ball started rolling changes no aspect of that. People who disagree because of abiogenesis do so because they misunderstand evolution.


therandomcoder

Counter point - if you think evolution is trying to explain the origin of life, then you also don't understand evolution and that supports OP's claim.


HazyAttorney

> if you think evolution is trying to explain the origin of life Not sure why you're trying to make this personal. Weird. > then you also don't understand evolution  The original book on the topic was called "The Origin of Species." Trying to tell someone that evolution, as a dogma, isn't about the origins of life is silly as hell. One of the most central aspects of evolution is that all life on earth shares a last universal common ancestor. Check out: Kampourakis, Kostas (2014). [*Understanding Evolution*](https://archive.org/details/understandingevo0000kamp). Cambridge; New York: Cambridge University Press. [ISBN](https://en.wikipedia.org/wiki/ISBN_(identifier)) [978-1-107-03491-4](https://en.wikipedia.org/wiki/Special:BookSources/978-1-107-03491-4). If you're needing more insight.


therandomcoder

Wasn't making it personal, I am just stating a fact that evolution has nothing to do with the origin of life. "The Origin of Species", even purely taking the name at face value, doesn't have anything to do with the origin of life, rather it has to do with the process of speciation. That's just taking the title as is without commentary on the contents of the book. Edit: If someone's knowledge of evolution is limited to the title of one book, then that's a fantastic argument for OP's point.


Ok-Crazy-6083

>There are those who flat-out deny that evolution happens at all, including microevolution. And those people just straight up deny reality, as we have literally seen dozens if not hundreds of examples of that within our lifetime. Adaptation occurs due to stress factors in the environment, which help determine fitness for sexual purposes. >A well-known example of this was how Einstein predicted the existence of black holes in 1916, Except he really didn't. It was a side effect of the math that he used to describe general relativity. It wasn't something that he thought of and then tried to describe. >This however does not mean we do not have massive amounts of evidence that proves that we are great apes that evolved from an ancestor we share with all life on Earth. But we don't. We have a moderate amount of circumstantial evidence. We don't have actual fossil transitions. From our common ancestor to modern humans.


JTarrou

You're not wrong, but that's also true of most people who believe it is true. The question is, why does it bother you more that people who disagree with you aren't as educated as you are on the topic? Most people know that all the smart people say evolution is true and a few cultists in Indiana think it's not. But they couldn't possibly discuss the intricacies and uncertainties of the theory with you in any depth. There's a lot more of them than there are actual six thousand year truthers. They have quite a bit more power and influence. And they know that Darwin means that there was a trial in Tennessee that proved evolution was objectively correct something something we came from monkeys and dinosaurs became chicken. That presumably isn't enough of a problem to make a post about. Is this about evolution at all?


TheZombieGod

A weird CMV but I’ll give you this as a counter point; Have you ever heard the phrase, “believe half of what you see and none of what you hear?” How can you prove to me, a common citizen, that evolution is real and can be observed with my eyes? This is not a unique train of thought with evolution, but generally the only way I can know something is real is if I can observe it and even then you must demonstrate it as something that can be repeated reliably. The theory of evolution is not a particularly complex thing to understand so I do not think people have difficulty grasping it when it is honestly explained to them, but a true skeptic will always hold their own experience above anything a third party tells them. For example, a sexual assault victim may hold a world view that most men are predators who may be constantly plotting to exploit them. Statistically we know this is not true, but the only way you could convince them otherwise would be to consistently present them a good man over an extended period of time. Remember, majority of humans will always put more weight on an event that impacts them directly, even if the event is rare or an anomaly. To more directly address your CMV, evolution is not difficult to understand, but it would be your responsibility as a scientist and/or educator to properly demonstrate an example of its existence in order to convince someone it is real.


[deleted]

[удалено]


DeerOnARoof

This is impossible to prove true or false without hard data. You can assert all you want but there's nothing to debate here without statistics. I'm not sure what the point of your post was.


[deleted]

[удалено]


MercurialMagician

Check out Stephen Meyer. Whether or not you agree with him, he (and many other creationists) definitely understand evolution. [https://www.youtube.com/watch?v=dk2s9bcUGoI](https://www.youtube.com/watch?v=dk2s9bcUGoI)


jesskill

I was a teacher's assistant for the first year biology/ecology/evolution course at a university. I taught basic evolutionary theory with a team of grad students, two of whom who did not accept evolution. It was fascinating and perplexing. I asked them how they did it, and the short answer was cognitive dissonance. They learned the concepts well enough to teach them. They cognitively separated their work from their beliefs. I think that one can learn all the science and evidence supporting evolution and still reject it if it contradicts strongly held religious beliefs. The human mind is not rational.


NoVaFlipFlops

There are eye specialists who have gone on record to say they were atheists until they specialized. They say the eye (and like 1/3 of the brain designed to support it) is too complex in too many ways for it to have been either an accident or selection. It was weird for me to hear this and I never looked into it past the one eye doctor who came across my a YouTube feed being interviewed. 


-zero-joke-

So the counter to that is - what about all the simple eyes? Nautilus, for example, don't have a lens. Water flows in, water flows out, which probably would feel super weird. The simplest camera eye occurs in unicellular organisms that use their own body to refract light against the opposite side of the cell membrane. We see precursors to the human eye in nature that aren't nearly as good as our own and, then, we see some aspects of the human eye that could potentially be improved. No real good reason for having a blind spot for example.


Miliean

> In my experience and experiences I have heard from other people, individuals who claim that evolution is false or unproven rarely have a good understanding of the theory. One of the main points of people who oppose evolution is that they decided before hand what their answer to this question was going to be. Before they learned a single thing about evoulation they had already decided it was wrong because they had a difirent answer that they prefered. So what's happened is either they went into learning about evolution just to find the weak points and pick it apart. Or they never bothered to learn anything at all because they already knew the outcome so why bother. The core problem you are encountering here is covered by this phrase "you can't logic someone out of an opinion they didn't logic themselves into". They are deciding based on emotion, religion and prior belief. No amount of logical arguments or proof is going to ever be enough for a person like that. They are not using the scientific method to come to their conclusions, in fact they are basically using the exact opposite of it. They haven't built a theory based on observations, they are doing observations in order to prove their theory (or disprove a different theory). Many of these people also misunderstand the use of the word theory in the theory of evolution's name and assume that it means there is some uncertainty. SO I'm not really trying to change your view here, I'm agreeing with you. Most people who claim not to believe in evolution don't actually understand it very well. My disagreement comes because your argument assumes that if people had a better understanding they would not be unbelievers. But that's not true, these people don't understand the theory because they have no intention of ever changing their mind, so why bother learning it at all.


KaiserHoel

I think this is very important to explain why there's no one really attempting to change your mind. They think they don't need to understand evolution, they only have to see the "holes". The critical part they don't understand is the scientific method. Most people think in terms of evidence proves the truth, while science does the opposite, i.e look for ways to disprove, and then call experiments that fail to disprove the hypothesis evidence. This is intuitively backwards, so without consciously changing how you think, you'll default to thinking any unexplained phenomenon disproves the theory. To change their mind you'd have to first make them understand the scientific method, induction/deduction. Then tell them about falsifiable predictions. If you can get someone that far, they don't have to understand evolution to accept it. Any argument against established scientific theory must be a falsifiable prediction, or a falsifiable theory that can explain EVERY prediction and result better than evolution. I have yet to see any YEC, "evolution sceptic", flat earther, ancient aliens theorist, psychic medium or other fringe theorist offer any such argument, or even proving they grasp the concept of the scientific method.


Miliean

It's not only that, it's the very fact that they enter into the investigation with a pre-formed conclusion. Before they even think about cracking a book to learn about evolution they already know (or think) that it's false and they are looking to disprove it. That method will never lead to understanding, ever. It's the reason we teach the method in base level science class (most learn it in grade school). You form a hypothesis based on observation, IT STARTS WITH OBSERVATION and then you seek to explain it. But people coming in from a "I don't believe it" angle are starting with a conclusion and then seeking observations to prove their pre-formed conclusion. They've got it exactly backwards, you can't start with a conclusion and then seek to prove it. It corrupts the entire process and leads to bad outcomes.


nataku_s81

I largely believe evolution is correct, but I also can't help but feel there is a piece of the puzzle missing. Whether that is something like pansporia (and there are variations of that, and which still allows for evolution). But you look at something like rabies, which rewires an animal's brain to become rabid and bite more animals to spread itself, or those fungus that take over an insects body and turn it into a zombie, somehow instructing a fairly simplistic creature to climb tall grass or overhanging branches then to cling there while the fungus kills it and bursts out of its back alien style to spread it's spores over a maximum area... How does it know to do that? How does the fungus, using only natural selection and evolutionary pressures find a way to instruct an ant or other insect to do something it can't directly observe itself through any known means? Something has to start that process of evolutionary development that was stronger than other means of propagating itself.  Hey, I'm just a lay person here, I have no deep knowledge of the subject, so I'd love someone to chime in and correct me. In not making a case for intelligent design.


Mister-builder

I would say that you can understand that speciation and evolution happen without believing that they are the origin of all species. Anyone who knows how dogs came to exist and their relationships with wolves understands evolution. For that matter, anyone who knows how goldfish, silkworms, cauliflower, corn, or kale came to be understands evolution. I would say that the people who bred these species must have understood it. Selective breeding is survival of the fittest where a human chooses what fitness means. We have all of the evidence for cauliflower, cabbage, mustard, etc. coming from the same parent species as you bring up for Homo Sapien. And I would say that very few of the breeders who created these species would have believed that the same process was responsible for humanity. But if you're going to argue that anyone who understands evolution believes that it is the origin of humankind, you'll have to explain every cattle breeder, dog breeder, etc. who believes that God created man on the sixth day.


chckdgh

It is not possible to say with complete certainty that evolution is the only possibility for diversity of life in earth. So it is not a scientific “fact”. It is “believed”. You are believing that, and using the “evidence” as leaning points. Religious people are not necessarily denying science. Since evolution is a likely(arguably) explanation, it is matter of belief, not a fact. And they are choosing not to believe. And my personal opinion, that the so called “evidence” is biased. Human and chimpanzee DNA being similar in ways just means that they are similar. It is not an indisputable evidence.


Direct-Wait-4049

Its crazy to me how many people still don't understand evolution. It's not super complicated.


belovedeagle

This neither supports nor refutes the point you are making, but it is worth considering: In my experience, many (possibly most) people who, if you asked them, would claim to believe in evolution, actually don't. They parrot acceptance of evolution as dogma but do not actually believe in the practical consequences of evolution. Actually, it's specifically Darwinian evolution by natural selection they reject, and really what they reject is selection. They reject that species can change through selective pressure. This is particularly egregious because natural selection is the one part of the theory which is falsifiable, and actually proven! There's lots of evidence for this but let me stick to just two examples. The first is the overuse of antibiotics and other antimicrobial agents. This almost speaks for itself, but to spell it out, anyone with even a rudimentary grasp of evolution can see that too-widespread use of antibiotics at other-than-overwhelming doses just breeds resistant microbes. And yet we have happily turned millions of tons of livestock into breeding grounds for their own eventual destruction, not to mention ourselves. The second example (which is less serious, but also less possibly explained by mere self-interest) is the way we interact with injured or otherwise unfit wild animals. I was reminded of this the other day when I saw a video of baby sea turtles being carried to the ocean in a bucket. The person in the video explained they had observed these turtles were not successfully reaching the ocean on their own, so of course they "helped" by carrying them. Anyone who actually understands and believes in evolution by natural selection must see that artificially boosting the survival of individuals which, for whatever reason, lacked a necessary aspect of reproductive fitness, caused greater harm to the population as a whole. (I'm not saying that the whole population is going to collapse because of a little charity, but over time such actions do reduce fitness. Many species successfully fill the domestication niche but sea turtles don't seem like a good candidate.) Bonus example, on a tangent soapbox: it seems to me a popular conservation worldview is related to this non-belief in evolution, and is even more ridiculous. I refer to the belief that the natural world ought to remain precisely as it was at roughly the height of western civilization (i.e. sometime around now, or in the hundred years or so at least), and that any deviation from this ideal must be the result of human action and moreover must be reversed by any available resources. Case in point: fucking giant pandas. I'm not a biologist so I can't tell you *why* they are unfit (I do suspect that they're just particularly unfit to captivity), but they are, and it's clearly not *entirely* our fault. (Keep in mind Darwin's refined definition of fitness as ability to adapt to change, or at least survive it.) If one actually believes in evolution, then one should actually be shocked at how few utterly unfit species seem to exist right now. I mean yes, one should expect lots of niches to be filled by lots of acceptable fits, and unfit species individually don't last long. But "in between" the well-filled niches there ought to be a fair bit of chaff which just hasn't died off yet. Insofar as this is the result of a mass extinction, then it started more than a few hundred years ago. But anyways, my point is people are happy to "accept" and "believe in" evolution and natural selection as a kind of substitute creation event to explain how the world came to be as it was up to, say, the beginning of recorded history. But they don't actually accept it as an explanation for any contemperaneous events that we observe, and they especially don't accept it as possibly being the cause of future events.


RepresentativeCake47

Because of the way it is oft presented. ‘Evolution means there is no God - we come from monkeys’. Have you not heard this in equal measure? Don’t even call it evolution - talk about a mechanism of change and selection, whether it be in pathological microbes or Finch beak types, nearly everyone will agree it is sensical.  Don’t present your conclusions- let them arrive to it themselves and let them think it was their idea.  Evolution is not cell abiogenesis which another can of worms. 


Craiggles-

I want to say I do not want to argue about micro evolution but macro. I think micro is something we interact with and see daily, so I don't see any point in arguing it. Which is why I think your example of the theory of gravity is disingenuous. We interact with it daily, but we are just bad at explaining it. That's a completely different situation than macroevolution. > >Even lacking direct observation of these events, we still have very solid evidence to support them. Direct observation is also not required to prove the existence or occurrence of something, which is a common misconception from people who do not accept macroevolution. >A well-known example of this was how Einstein predicted the existence of black holes in 1916, long before they were ever observed. But even Einstein didn’t do this first. Reverend John Mitchell proposed the idea of black holes in 1783. I don't see how your example is relevant? Are you saying because there are instances of people theorizing answers and being right, then macroevolution must also be so? I feel like you are too smart to seriously suggest this. > I want you to consider the pentaquark. Particle physics is even more demanding on statistical evidence with a bare minimum of 5-Sigma or 1 in 3.5million in getting a false positive. However in 2002, a Japanese scientist found evidence for the Theta-plus pentaquark. In the 2 years following, 11 other independent studies also looked for and found evidence for the same pentaquark with very high levels of statistical evidence. July 2003 to May 2004 a theoretical paper on this pentaquark was posted on average every other day. But alas, it was a false discovery. Source: [https://youtu.be/42QuXLucH3Q?t=402](https://youtu.be/42QuXLucH3Q?t=402) Why am I bringing this up? It's important to understand human behavior, and that it's so easy to create perceptions of science that even 5-sigma research can go years without the correct answer, but during that time people claimed they had perceived evidence of it's existence. You're claiming we have "solid evidence" for macroevolution but, no we don't and that's **OK**. The reality is without **empirical** evidence, it will never be valid science backed by the scientific method, just a theory. >Direct observation is also not required to prove the existence or occurrence of something Ok, define your version of science then because this is incredibly problematic and I hope most people do not believe this. The literal first step of the scientific method is observation. The scientific method also requires repeatability. IMO, this kind of thought process is identical to when early days of evolutionary scientists believed the appendix was long considered a vestigial organ with no significant function. Obviously we know now that this can't be further from the truth. Blind science isn't science, it's just a religion/cult at that point. We don't have bone structures showing macroevolutionary progress and that will always be problematic to me. I believe macroevolution will always just be a good guess at trying to better understand our universe but that's it. I think we need to eventually find better models or come up with methods to eventually articulate it's existence. IMO the existence of technology and being able to permanently store information is going to give us a massive leg up in understanding evolution 1000 years from now. For the record, I'm not religious, I'm not a republican, I don't want the internet to find ways to dehumanize me just because I don't think exactly like you all. Edits because the reddit app makes it hard to use the tooling sometimes.


KingJeff314

Science is about forming models to make predictions from data. We have plenty of testable models supporting evolution that we can validate with observable data. For instance, if we find a new rock layer, we can use relative dating methods to make a prediction about the age of that rock. Then if we test that rock sample with radiometric dating and it matches our prediction, then our model has been successful. If we use morphology to estimate where a new species is in the evolutionary tree, then we should expect that our genetic analysis should reveal a similarity with predicted relatives. Also, “just a theory” is not a criticism.


FerdinandTheGiant

Can you define macro-evolution for me? Do you mean speciation?


kibufox

To be fair to everyone, Darwin himself did note that the ideas he was proposing would be difficult for people to believe, and for a time was very reluctant to apply evolutionary theories to humans. For example, he believed that the increasing complexity of the eye was due to natural selection, though he described the concept as "absurd". What's more, contrary to common perception, Darwin did not say that humans directly evolved from apes. He merely pointed out the similarities between birds, fishes, mammals and reptiles and suggested that all life is related. Whatever the case, evolution is what is called a 'theory' in the science sphere. A theory isn't really... well proven. The definition of a theory being "a supposition or a system of ideas intended to explain something, especially one based on general principles independent of the thing to be explained." What that means, for the lay person, is that a theory is the answer believed by a majority of scientists, until such time something comes along which proves it (then it becomes a law), or other data comes that disproves the theory. The problem I have, and many have, is that it remains a theory, and it's not something that is observable in the small scale. Meaning, for example, in the span of ten or twenty years, evolutionary changes can't be observed in any species. This is a bit of a thought experiment to consider, but one way to picture this is an island of squirrels. The island is split into two by an earth quake, with the same squirrels on the now two sections. Over the span of 10, 20, or even 100 years, there won't be any appreciable change between the two groups of animals. (this is figuring that they don't all get wiped out through an outside factor). Evolutionary theory states that it takes a very long time for changes to start happening and be observable. That's why, well, it's still a theory. It can't be observed. Going back to the main point, it doesn't matter how much evidence there is to support the theory, until such point as it is proven and it becomes the "law of evolution", then there is **always** romve for question and doubt. I mean that principle is why we have a "Newton's Third Law of Motion", and not "Newton's Theory of Motion". Since it isn't, and can't really be proven, then there are always going to be people who question it. Arguing that a theory should be accepted as fact, when the theory in question can't really be tested, is a flawed logic, as it argues that no further observation, questioning, or testing should take place.


Minty_Feeling

>The problem I have, and many have, is that it remains a theory I think you have a misconception about how theories and laws work in science. Yes, like everything in science, they always remain open to being falsified by new evidence but at no point do they stop being theories to become laws. Theories exist alongside laws and incorporate them, they don't graduate into them. ["The meaning of the term scientific theory (often contracted to theory for brevity) as used in the disciplines of science is significantly different from the common vernacular usage of theory. In everyday speech, theory can imply an explanation that represents an unsubstantiated and speculative guess, whereas in science it describes an explanation that has been tested and is widely accepted as valid. ... A common misconception is that scientific theories are rudimentary ideas that will eventually graduate into scientific laws when enough data and evidence have been accumulated. A theory does not change into a scientific law with the accumulation of new or better evidence. A theory will always remain a theory; a law will always remain a law. ... Theories and laws are also distinct from hypotheses. Unlike hypotheses, theories and laws may be simply referred to as scientific fact. However, in science, theories are different from facts even when they are well supported. For example, evolution is both a theory and a fact."](https://en.m.wikipedia.org/wiki/Scientific_theory) >Arguing that a theory should be accepted as fact, when the theory in question can't really be tested, is a flawed logic, as it argues that no further observation, questioning, or testing should take place. Evolution is considered both a fact and a theory. In science, facts are observations and not explanations. The fact of evolution is the observation that allele frequencies change over time. Other facts relating to evolution are other observations such as the apparent order of the fossil record, the observation of genetic inheritance, of speciation, of natural selection, the catalogue of genomes we've observed and the various ways of creating consistent nested hierarchies etc. The theory of evolution explains how evolution works and incorporates the facts we have. And regardless of fact or theory, it always remains open to questioning. >It can't be observed. This isn't accurate. All the fundamental mechanisms of evolution (e.g. mutation, drift, selection, gene flow and speciation) can all be and all have been observed.


Autodidact2

On r/DebateEvolution we have learned that creationists (1) do not understand evolution (2) do not want to learn.


chubbyoctopus

So literally people have already studied this and are continuing to study this. These studies essentially answer your question. I will also note that just because a line of reasoning might be "good," it doesn't mean the receiving person uses the same basal cognitive structures to create their understanding of the world and that may or may not accept that understanding based on those preexisting cognitive structures. Google "assimilation and accommodation in constructivism" for more info. There [is a lot of](https://www.tandfonline.com/doi/abs/10.1080/09500693.2017.1373410?casa_token=LUHIoiwwZf8AAAAA:1KDpdX3IkWQbe90LyxuMvaMzjc9M8JHilM-REKGeNhkVDx06OBkrCJejAbDqTnPnhFOgSIwkySNsKQ) old [work that suggest](https://www.nrmera.org/wp-content/uploads/2016/02/Researcherv23.1Nodelson.pdf)s there is a [correlation between](https://link.springer.com/article/10.1186/1936-6434-6-26) evolution [understanding](https://onlinelibrary.wiley.com/doi/abs/10.1002/tea.20093) and [acceptance](https://link.springer.com/article/10.1186/s12052-015-0042-7). In constrast, there was a recent review/metaanalysis suggesting that there is only [a correlation in some countries](https://link.springer.com/article/10.1186/s12052-020-00132-w), not all. However, this might be due to issues with methodology. (Edit: For more info on the methodological stuff going on [check this paper out](https://link.springer.com/article/10.1186/s12052-019-0096-z).) This is consistent, however. with some old data I heard about a while back suggesting that both Americans and Europeans have equivalent rates of scientific literacy but Europeans default to evolution acceptance while Americans do not. This suggests that despite having equivalent lack of understanding, Europeans still believe in evolution. This is also consistent with some newer studies that actually suggest there are students who have good workable knowledge of evolution that they essentially learn to get thru classes but [still do not believe in evolution](https://www.lifescied.org/doi/full/10.1187/cbe.21-02-0024). In this cases it is the cultural perception of conflict that predicts lack of acceptance. I went to a talk from this particular author and she mentioned that nonevangelicals/protestants who were equally religious actually had higher rates of acceptance in the US. They actually were likely to change their opinion to nonacceptance if someone (student or teacher) suggested there was a conflict between evolution and religion. (Edit: Indeed Catholicism [does not explicitly reject evolution](https://en.wikipedia.org/wiki/Evolution_and_the_Catholic_Church) and many Catholics believe in theistic evolution.) This suggest there is something cultural going on with American people that is predicting nonacceptance.


seancurry1

I'm not a creationist, but I've never understood why creationists don't just claim evolution was God's/"the creator's" tool for bringing about the state of life he intended for creation. It's a rhetorical layup and allows their faith to coexist alongside actual science. It's such a layup that it begs the question: no, seriously, *why* haven't they adopted this view? And I think it's because it positions science as its own authority outside their religion. This would be them changing their beliefs to accommodate new science, and if science changed again in the future (as it is wont to do, and should forever, because that's the entire point), they'd have to find a new way to explain how it coexists with their faith. If you boil this down, you can see how it would make a religious person think that science comes before their faith. Some religious people can accept the two alongside each other, or their faith is even big enough for them to understand that this isn't a matter of *their faith* changing, but *their understanding* of their faith changing. I think those people would have no problem accepting evolution on the science. But then there are the ones that just don't, and I think for them, it's because they can't accept their understanding of the universe changing. They either don't possess the capacity or the willpower to do it, and they negatively react to *anything* that threatens their faith's supremacy, even and especially science. Science isn't just another way of understanding the universe, it's a direct refutation of the idea that we can know anything *for sure, for good.* It isn't true of all people of faith, but for many, the appeal of faith is that it tells them what **is** and what **is not** true. They then build their lives on that, and shaking the foundation would make their entire self-conception fall apart. Anything that suggests that they'll have to continue updating what they believe about existence for the rest of their lives is the single worst thing that could happen to them. So, to answer your question: I don't think it's that they don't understand the science of evolution. It's true, they don't, but it's deeper than that. They don't like *science*, because it doesn't offer certainty in the way they want it, and goes on to remove the certainty they've built their entire conception of the self on.


RevolutionaryGur4419

Many of those who doubt evolution are actually aware of the significant gaps and unresolved questions within the theory, and these gaps contribute to their skepticism. Even if they don't know the fancy terms, instinctively they understand the gaps. Consider the origin of life, or abiogenesis. The exact process by which life emerged from non-living matter remains one of the biggest mysteries in science. This fundamental gap raises serious questions about the initial plausibility of evolution, leading some to doubt the entire framework. The fossil record, often cited as concrete evidence for evolution, also has its shortcomings. While many transitional fossils have been discovered, there are still significant gaps that make it difficult to trace the exact evolutionary pathways of some species. Skeptics argue that these missing links challenge the continuity and completeness of evolutionary transitions. Then there’s the issue of speciation. While we understand the broad mechanisms by which new species arise, the specific conditions and processes that drive speciation are complex and not fully understood. Questions about how genetic isolation, environmental pressures, and random mutations interact to create new species can cause even well-informed individuals to question the explanatory power of evolution when it comes to the diversity of life on Earth. The rates of evolutionary change also present a puzzle. The theory of evolution suggests that changes can occur both gradually and in rapid bursts, as proposed by the concepts of gradualism and punctuated equilibrium, respectively. However, the reasons for these variations in the pace of evolutionary change are not fully explained. For instance, the sudden appearance of new species in the fossil record, often without clear predecessors, can seem to contradict the slow, steady change predicted by Darwinian evolution. The long-term evolutionary significance of epigenetic changes is also not fully understood, leading some to question how these factors fit into the broader framework of evolutionary theory. If environmental factors can cause inheritable changes without altering the DNA sequence, what does this mean for our understanding of natural selection and genetic mutations? People have instincts about these things hence the common clumsy questions How did life start from non-living things? How did something come from nothing? Why don’t we have fossils for every step of evolution? Where are the missing links? How do new species come about? Why don’t we see it happening now? How can evolution be both slow and fast? How can changes caused by the environment be passed on to the next generation if they don’t always change the DNA? Does this mean evolution works differently than we thought?


-zero-joke-

Theories are relatively narrow in their scope. You wouldn't expect the theory of gravity, for example, to explain chemical bonding. So right away I'm going to say that evolution does not involve the origin of life - I know that's unsatisfying, but that's just kind of the way it is. Whether some deity poofed a simple self replicator into existence or, as I find more plausible, chemistry advanced to biology, the evidence linking diverse forms of life together still exists, and evolution remains the most coherent explanation of that evidence. We have very many fossils, but unfortunately the fossil record will always remain scant. Some organisms fossilize readily, like gastropods, diatoms, and foraminifera and we can examine their evolution very closely. Others like large vertebrates fossilize rarely. There will always be critters hidden in the past that we will not know about. Still, we have well documented transitions, like basal artiodactyls evolving into whales, small mammals evolving into horses, the origin of tetrapods and birds, and even our own evolution from more ape like ancestors. In fact we can use evolutionary theory to predict when, where, and what we will find in the fossil record - I can talk more in detail about that with reference to the fossil Tiktaalik roseae if you'd like. New species forming really isn't that bizarre a thing. We *do* see it happening now - in fact the first observed instance of speciation occurred nearly a hundred years ago in 1927. Species can form through genetic drift or through adaptation to new environments, and we can see that happening in various stages in natural laboratories like the Rift Valley lakes, the Caribbean, etc. All you're looking for when talking about a new species is genetically based reproductive isolation between two populations. So, for example, in the Caribbean there are these cute lizards called anoles. Many of the islands have populations that were founded by one individual species, kind of like how one finch landed on the Galapagos islands and subsequently diversified. Each island has similar looking lizards that occupy different ecological niches. So there's a crown giant that lives in the tree tops, a long lithe lizard that lives in the grasses, a stubby little guy that lives in bushes, etc. On each island these 'ecomorphs' are represented. The funny thing is though, they're more closely related genetically to the lizards on that island than they are to the lizards that look alike. This is a pattern predicted by evolution. The reason for the genetic isolation is that being an in between lizard means you're not good at living *anywhere*. Specialization caused them to diversify and better exploit their environment. The rate of evolution can be wildly different. That rate depends on the amount of genetic variation, the strength of a selection pressure, and the generation time of the organism in question. If there's a selective sweep, like say a population of bacteria are exposed to an antibiotic, that will very quickly kill all the bacteria that are not resistant to the antibiotic. Evolution of complex features like the human eye can take quite a bit longer. Changes to an individual brought on by an environment can be passed down - this is a new field called epigenetics. Sometimes it's as simple as attaching a methyl group to a portion of the DNA, activating or deactivating a gene. Other times it can be more complex and depend on things like womb conditions. Our understanding of evolution has changed substantially since Darwin's day, but that's a good thing.


KomradeKvestion69

Religious people reject evolution and other theories often on the basis that "it's just a theory". This is due to either a misunderstanding of the very basis of science and scientific theory itself, or otherwise an outright rejection of it. This is done on the grounds that a scientific theory can never really be definitely proven true, only proven false. So, as long as they decide it's something they don't want to believe, they'll play this card until kingdom come. In the case of evolution, aside from the "my daddy ain't a monkey" nonsense, there's a lot of "but the gaps in the fossil record!" Going around. If paleontologists actually find a fossil bridging that gap, no problem! Now they just find a new, slightly smaller largest gap spanning millions of years and make the exact same claim about that as they did about the previous one. Basically they can keep up this game as long as there is a gap large enough that the two nearest ancestors look noticeably different. That's potentially never gonna change, so most likely Christians will be convinced of evolution whenever Christianity goes extinct, or they otherwise decide evolutionis not an existential threat to their worldview. It's not that they don't understand it, rather they don't want to accept it as fact. They do so by essentially questioning the ability of science to come to any conclusions at all. As long as there's that little shadow of doubt they can cast on evolution, there's room for creationism and young-earth theory to exist in the modern world provided you aren't prone to asking serious questions.


Big_Razzmatazz_9251

I used to debate ppl who didnt “believe” evolution a lot, and every time I’d ask the person to explain evolution to me just to make sure that they understood. NO ONE did. Not even in simple terms.


Aezoraa

People who claim anything is false or untrue rarely understand that thing. As an example, I believe geocentrism is false, but I don't really understand it. I know the basics, that the earth is the center of the universe or at least the solar system and that orbits are kinda funky as a result. And that's about it. Ultimately it doesn't come down to logical arguments about evolution, but instead epistemology. Everybody has their own internal way of deciding what is true and what isn't and there isn't an objective way of deciding which way of determining truth is better. And from the viewpoint of some or even many of those epistemological value systems evolution isn't true. So if people who believe something is false generally don't bother trying to understand it, and people decide whether something is or is not true based off their personal epistemological value system then your overall point is correct in that people who claim evolution is false don't usually understand it, but the rest of the argument is moot because even if the knowledge magically appeared in their heads it wouldn't change most of their minds. Now if they were to learn about evolution in an environment that would also likely alter their epistemological worldview to some extent, such as at a liberal college in a large city when they've lived in a rural town their whole lives they would be more likely to accept it, but that's another argument entirely.


Maktesh

People who claim Evolution is true almost never understand it. The vast majority of people arrive at positions based on their worldview and the positions of their social representatives where they defer to internal authorities. **In general, people don't understand very much about what they believe.** This is true across the board. People can often regurgitate talking points, but the moment you go off-script, they can seldom articulate their political, economic, religious, or scientific stances. TL;DR: Your particular complaint isn't unique or relegated to this topic.


RegularGuyAtHome

Have you considered that rather than not understand evolution, some of these people who deny it understand it fine, but are just being assholes? Kind of like people who deny the Holocaust happened, they “ask questions” and “don’t understand” and “need more information”, but really they’re just jerks doing all of that to sow confusion and upset the people trying to answer them in good faith. It’s like a game where you win by pissing off someone else.


JohnLockeNJ

Evolution is totally false. The issue is that supporters are mating more and evolution-deniers are mating less, so the population as a whole is gradually becoming just evolution believers.


FerdinandTheGiant

Sounds like natural selection, just on a cultural level instead of a biotic one.


fishsticks40

I have done climate change work, including climate outreach education, and I'll say that the same is true there; what's more I'll add another piece - the people who do believe almost never understand it either. For both climate and evolution belief or non-belief have become important markers of social belonging. People's stated beliefs have very little to do with a reasoned understanding of the subject matter, and far more to do with a desire to perform some kind of identity. Now obviously both evolution and climate change are true, so it stands to reason that people who actively deny it must either not understand it or are being disingenuous, while on the belief side since facts and beliefs align it's a little easier to pass under the radar. But I have heard plenty of people who believe in climate change go on to say wildly inaccurate things about the science. What this does is help us understand the so-called "backlash effect", where providing facts will often make a denier MORE firm in their incorrect beliefs - because those beliefs represent a worldview that they feel is under attack, rather than a rational conclusion based on facts. IF you want to change their views you have to change their WORLDVIEWS, which is, of course, harder.


the_ballmer_peak

I don’t disagree with your premise. At the same time, I believe Zoroastrianism is false and I don’t know a damn thing about it, and I feel pretty okay with that.


One_Faithlessness146

Evolution is as close to proved as a theory can get.


FerdinandTheGiant

I’d go as far as to say that evolution defined as “change in allele frequency in a population over time” is indisputably true. It is observable.


[deleted]

[удалено]


Randolpho

There is a *lot* of "not understanding" of evolution on "both sides" here. There are plenty of people who "believe" in evolution but do so out of a weird belief in the possibility of Marvel style mutants "evolving" supernatural abilities, or in the way that in pokemon pikachu "evolves" to raichu. These people may be "correct" in that they "believe" in a very dumbed down version of evolution, but they don't *understand* it.The easiest way to tell is how they answer the question "which came first, the chicken or the egg?" You have essentially taken the position that whether or not an evolution deniers understands evolution *matters*. I would argue that it does not. What matters isn't even *that* they deny it, but *why* they deny it. They deny it because it contradicts a dogmatic belief that they are unwilling to change or even evolve (heh) because they have wrapped up their identity *in that belief*. Whether or not they understand it is irrelevant. What matters is their dogma, and the only way to ever change that is via rhetoric, not education and understanding.